Está en la página 1de 19

STANDARD OIL COMPANY V JARAMILLO The Power of the Registry of Deeds is Ministerial, and The absolute criterion to determine

between real and personal property is NOT supplied by the civil code. Parties may agree what to treat as personal property and what to treat as real property. FACTS On November 27, 1922, Gervasia de la Rosa was the lessee of a parcel of land situated in the City of Manila and owner of the house of really tough materials built thereon. She executed that fine day a document in the form of a chattel mortgage, purporting to convey to Standard Oil Company of New York (by way of mortgage) both the leasehold interest in said lot and the building. After said document had been duly acknowledged and delivered, Standard Oil presented it to Joaquin Jaramillo, as register of deeds of the City of Manila, for the purpose of having the same recorded in the book of record of chattel mortgages. Upon examination of the instrument, Jaramillo opined that it was not chattel mortgage, for the reason that the interest therein mortgaged did not appear to be personal property, within the meaning of the Chattel Mortgage Law, and registration was refused on this ground only. Later this confusion was brought to the Supreme Court upon demurrer by Joaquin Jaramillo, register of deeds of the City of Manila, to an original petition of the Standard Oil Company of New York, demanding a mandamus to compel the respondent to record in the proper register a document purporting to be a chattel mortgage executed in the City of Manila by Gervasia de la Rosa, Vda. de Vera, in favor of the Standard Oil Company of New York. The Supreme Court overruled the demurrer, and ordered that unless Jaramillo interposes a sufficient answer to the petition for mandamus by Standard Oil within 5 days of notification, the writ would be issued as prayed, but without costs. ISSUE: w/n the Registry of Deeds can determine the nature of property to be registered. w/n the Registry of Deeds has powers beyond Ministerial discretion. RESOLUTION: 1.Jaramillo, register of deeds, does not have judicial or quasi-judicial power to determine nature of document registered as chattel mortgage Section 198 of the Administrative Code, originally of Section 15 of the Chattel Mortgage Law (Act 1508 as amended by Act 2496), does not confer upon the register of deeds any authority whatever in respect to the "qualification," as the term is used in Spanish law, of chattel mortgages. His duties in respect to such instruments are ministerial only. The efficacy of the act of recording a chattel mortgage consists in the fact that it operates as constructive notice of the existence of the contract, and the legal effects of the contract must be discovered in the instrument itself in relation with the fact of notice. 2.Article 334 and 335 of the Civil Code does not supply absolute criterion on distinction between real and personal property for purpose of the application of the Chattel Mortgage Law Article 334 and 335 of the Civil Code supply no absolute criterion for discriminating between real property and personal property for purposes of the application of the Chattel Mortgage Law. Those articles state rules which, considered as a general doctrine, are law in this jurisdiction; but it must not be forgotten that under given conditions property

may have character different from that imputed to it in said articles. It is undeniable that the parties to a contract may be agreement treat as personal property that which by nature would be real property; and it is a familiar phenomenon to see things classed as real property for purposes of taxation which on general principle might be considered personal property. Other situations are constantly arising, and from time to time are presented to the Supreme Court, in which the proper classification of one thing or another as real or personal property may be said to be doubtful.] Sergs Products, Inc. vs. PCI Leasing G.R. No. 137705. August 22, 2000 FACTS: PCI Leasing and Finance filed a complaint for sum of money, with an application for a writ of replevin. Judge issued a writ of replevin directing its sheriff to seize and deliver the machineries and equipment to PCI Leasing after 5 days and upon the payment of the necessary expenses. The sheriff proceeded to petitioner's factory, seized one machinery, with word that he would return for other machineries. Petitioner (Sergs Products) filed a motion for special protective order to defer enforcement of the writ of replevin. PCI Leasing opposed the motion on the ground that the properties were still personal and therefore can still be subjected to seizure and writ of replevin. Petitioner asserted that properties sought to be seized were immovable as defined in Article 415 of the Civil Code. Sheriff was still able to take possession of two more machineries In its decision on the original action for certiorari filed by the Petitioner, the appellate court, Citing the Agreement of the parties, held that the subject machines were personal property, and that they had only been leased, not owned, by petitioners; and ruled that the "words of the contract are clear and leave no doubt upon the true intention of the contracting parties." ISSUE: Whether or not the machineries became real property by virtue of immobilization. Ruling: Petitioners contend that the subject machines used in their factory were not proper subjects of the Writ issued by the RTC, because they were in fact real property. Writ of Replevin: Rule 60 of the Rules of Court provides that writs of replevin are issued for the recovery of personal property only. Article 415 (5) of the Civil Code provides that machinery, receptacles, instruments or implements intended by the owner of the tenement for an industry or works which may be carried on in a building or on a piece of land, and which tend directly to meet the needs of the said industry or works In the present case, the machines that were the subjects of the Writ of Seizure were placed by petitioners in the factory built on their own land.They were essential and principal elements of their chocolate-making industry.Hence, although each of them was movable or personal property on its own, all of them have become immobilized by destination because they are essential and principal elements in the industry. However, contracting parties may validly stipulate that a real property be considered as personal. After agreeing to such stipulation, they are consequently

estopped from claiming otherwise.Under the principle of estoppel, a party to a contract is ordinarily precluded from denying the truth of any material fact found therein. Section 12.1 of the Agreement between the parties provides The PROPERTY is, and shall at all times be and remain, personal property notwithstanding that the PROPERTY or any part thereof may now be, or hereafter become, in any manner affixed or attached to or embedded in, or permanently resting upon, real property or any building thereon, or attached in any manner to what is permanent. The machines are personal property and they are proper subjects of the Writ of Replevin NAVARRO V. PINEDA FACTS: Pineda and his mother executed real estate and chattel mortgages in favor of Navarro, to secure a loan they got from the latter. The REM covered a parcel of land owned by the mother while the chattel mortgage covered a residential house. Due to the failure to pay the loan, they asked for extensions to pay for the loan. On the second extension, Pineda executed a PROMISE wherein in case of default in payment, he wouldnt ask for any additional extension and there would be no need for any formal demand. In spite of this, they still failed to pay. Navarro then filed for the foreclosure of the mortgages. The court decided in his favor. HELD: Where a house stands on a rented land belonging to another person, it may be the subject matter of a chattel mortgage as personal property if so stipulated in the document of mortgage, and in an action by the mortgagee for the foreclosure, the validity of the chattel mortgage cannot be assailed by one of the parties to the contract of mortgage. Furthermore, although in some instances, a house of mixed materials has been considered as a chattel between the parties and that the validity of the contract between them, has been recognized, it has been a constant criterion that with respect to third persons, who are not parties to the contract, and specially in execution proceedings, the house is considered as immovable property. BICERRA V. TENEZA FACTS: The Bicerras are supposedly the owners of the house (PhP 20,000) built on a lot owned by them in Lagangilang, Abra; which the Tenezas forcibly demolished in January 1957, claiming to be the owners thereof. The materials of the house were placed in the custody of the barrio lieutenant. The Bicerras filed a complaint claiming actual damages of P200, moral and consequential damages amounting to P600, and the costs. The CFI Abra dismissed the complaint claiming that the action was within the exclusive (original) jurisdiction of the Justice of the Peace Court of Lagangilang, Abra. The Supreme Court affirmed the order appealed. Having been admitted in forma pauperis, no costs were adjudged.

ISSUE: Whether or not the house is immovable property even if it is on the land of another RULING: Yes. House is immovable property even if situated on land belonging to a different owner; Exception, when demolished. A house is classified as immovable property by reason of its adherence to the soil on which it is built (Article 415, paragraph 1, Civil Code). This classification holds true regardless of the fact that the house may be situated on land belonging to a different owner. But once the house is demolished, as in this case, it ceases to exist as such and hence its character as an immovable likewise ceases. ASSOCIATED INSURANCE AND SURETY COMPANY V. IYA, ET. AL 103 SCRA 972 FACTS: Spouses Valino were the owners of a house, payable on installments from Philippine Realty Corporation. To be able to purchase on credit rice from NARIC, they filed a surety bond subscribed by petitioner and therefor, they executed an alleged chattel mortgage on the house in favor of the surety company. The spouses didnt own yet the land on which the house was constructed on at the time of the undertaking. After being able to purchase the land, to be able to secure payment for indebtedness, the spouses executed a real estate mortgage in favor of Iya. The spouses were not able to satisfy obligation with NARIC, petitioner was compelled to pay. The spouses werent able to pay the surety company despite demands and thus, the company foreclosed the chattel mortgage. It later learned of the real estate mortgage over the house and lot secured by the spouses. This prompted the company to file an action against the spouses. Also, Iya filed another civil action against the spouses, asserting that she has a better right over the property. The trial court heard the two cases jointly and it held that the surety company had a preferred right over the building as since when the chattel mortgage was secured, the land wasnt owned yet by the spouses making the building then a chattel and not a real property. HELD: A building certainly cannot be divested of its character of a realty by the fact that the land on which it is constructed belongs to another. To hold it the other way, the possibility is not remote that it would result in confusion, for to cloak the building with an uncertain status made dependent on ownership of the land, would create a situation where apermanent fixture changes its nature or character as the ownership of the land changes hands. In the case at bar, as personal properties may be the only subjects of a chattel mortgage, the execution of the chattel mortgage covering said building is null and void. Lopez v. Orosa LOPEZ V. OROSA AND PLAZA THEATREG.R. Nos. L-10817-18 February 28, 1958 FACTS:

-Petitioner Lopez was engaged in doing business under the trade name LopezCastelo Sawmill. Orosa, a resident of the same province as Lopez, invited the latter to make an investment in the theatre business. Lopez declined to invest but agreed to supply the lumber necessary for the construction of the proposed theatre. They had an oral agreement that Orosa would be personally liable for any account that the said construction might incur and that payment would be on demand and not cash on delivery basis. Lopez delivered the which was used for construction amounting to P62,255.85. He was paid only P20,848.50, leaving a balance of P41,771.35. The land on which the building was erected previously owned by Orosa, was later on acquired by the corporation. . As Lopez was pressing Orosa for payment, the latter and president of the corporation promised to obtain a bank loan by mortgaging the properties of the Plaza Theatre., out of which the unpaid balance would be satisfied. But unknown to Lopez, the corporation already obtained a loan with Luzon Surety Company as surety, and the corporation in turn executed a mortgage on the land and building in favor of the said company as counter-security. Due to the persistent demands of Lopez, Orosa executed a deed of assignment over his shares of stock in the corporation. As it remained unsettled, Lopez filed a case against Orosa and Plaza theatre praying that they be sentenced to pay him jointly and severally of the unpaid balance; and in case defendants fail to pay, the land and building owned by the corporation be sold in public auction with the proceeds be applied to the balance; or the shares of stock be sold in public auction. The lower court held that defendants were jointly liable for the unpaid balance and Lopez thus acquired the material mans lien over the construction. The lien was merely confined to the building and did not extend to the on which the construction was made. Lopez tried to secure a modification of the decision, but was denied. ISSUES: Whether the material mans lien for the value of the materials used in the construction of the building attaches to said structure alone and doesnt extend to the land on which the building is adhered to. Whether the lower court and CA erred in not providing that the material mans liens is superior to the mortgage executed in favor of surety company not only on the building but also on the land. HELD: -The material mans lien could be charged only to the building for which the credit was made or which received the benefit of refection, the lower court was right in, holding at the interest of the mortgagee over the land is superior and cannot be made subject to the material man's lien.

-Generally, real estate connotes the land and the building constructed thereon, it is obvious that the inclusion of the building in the enumeration of what may constitute real properties could only mean one thingthat a building is by itself an immovable property. -In the absence of any specific provision to the contrary, a building is an immovable property irrespective of whether or not said structure and the land on which it is adhered to belong to the same owner. -The law gives preference to unregistered refectionary credits only with respect to the real estate upon which the refectionary or work was made. - The lien so created attaches merely to the immovable property for the construction or repair of which the obligation was incurred. Therefore, the lien in favor of appellant for the unpaid value of the lumber used in the construction of the building attaches only to said structure and to no other property of the obligors. TUMALAD V. VICENCIO 41 SCRA 143 FACTS: Vicencio and Simeon executed a chattel mortgage in favor of plaintiffs Tumalad over their house, which was being rented by Madrigal and company. This was executed to guarantee a loan, payable in one year with a 12% per annum interest. The mortgage was extrajudicially foreclosed upon failure to pay the loan. The house was sold at a public auction and the plaintiffs were the highest bidder. A corresponding certificate of sale was issued. Thereafter, the plaintiffs filed an action for ejectment against the defendants, praying that the latter vacate the house as they were the proper owners. HELD: Certain deviations have been allowed from the general doctrine that buildings are immovable property such as when through stipulation, parties may agree to treat as personal property those by their nature would be real property. This is partly based on the principle of estoppel wherein the principle is predicated on statements by the owner declaring his house as chattel, a conduct that may conceivably stop him from subsequently claiming otherwise. In the case at bar, though there be no specific statement referring to the subject house as personal property, yet by ceding, selling or transferring a property through chattel mortgage could only have meant that defendant conveys the house as chattel, or at least, intended to treat the same as such, so that they should not now be allowed to make an inconsistent stand by claiming otherwise. Valdes v. Central Altagracia, Inc., 225 U.S. 58 (1912) Valdes v. Central Altagracia, Incorporated Nos. 193, 196 Submitted March 6, 1912 Decided May 13, 1912 225 U.S. 58 Syllabus

The record in this case shows that the court below did not err in bringing this case to a speedy conclusion and avoiding the loss occasioned by the litigation to all concerned. A litigant cannot, after all parties have acquiesced in the order setting the case for trial and the court has denied his request for continuance, refuse to proceed with the trial on the ground that the time to plead has not expired, and when such refusal to proceed is inconsistent with his prior attitude in the case. The granting of a continuance is within the sound discretion of the trial court, and not subject to be reviewed on appeal except in cases of clear error and abuse; in this case, the record shows that the refusal to continue on account of absence of witness was not an abuse, but a just exercise, of discretion. Under the circumstances of this case, and in view of the existence of an equity of redemption under prior transfers, held that a transfer of all the property of a corporation to one advancing money to enable it to continue its business was not a conditional sale of the property, but a contract creating security for the money advanced, and, on liquidation of the assets, the transferee stood merely as a secured creditor The mere form of an instrument transferring property of a debtor cannot exclude the power of creditors to inquire into the reality and substance of a contract unrecorded, although required by law to be recorded in order to be effective against third parties. Under the general law of Porto Rico, machinery placed on property by a tenant does not become immobilized; when, however, a tenant places it there pursuant to contract that it shall belong to the owner, it becomes immobilized as to that tenant and his assigns with notice, although it does not become so as to creditors not having legal notice of the lease. In this case, held that the lien of the attachment of a creditor of the tenant on machinery placed by the tenant on a sugar Central in Porto Rico is superior to the claim of the transferee of an unrecorded Davao Sawmill Co. vs Castillo 61 PHIL 709 GR No. L-40411 August 7, 1935 A tenant placed machines for use in a sawmill on the landlord's land. FACTS Davao Sawmill Co., operated a sawmill. The land upon which the business was conducted was leased from another person. On the land, Davao Sawmill erected a building which housed the machinery it used. Some of the machines were mounted and placed on foundations of cement. In the contract of lease, Davo Sawmill agreed to turn over free of charge all improvements and buildings erected by it on the premises with the exception of machineries, which shall remain with the Davao Sawmill. In an action brought by the Davao Light and Power Co., judgment was rendered against Davao Sawmill. A writ of execution was issued and the machineries placed on the sawmill were levied upon as personalty by the sheriff. Davao Light and Power Co., proceeded to purchase the machinery and other properties auctioned by the sheriff. ISSUE Are the machineries real or personal property? HELD Art.415 of the New Civil Code provides that Real Property consists of: (1) Lands, buildings, roads and constructions of all kinds adhered to the soil; xxx (5) Machinery, receptacles, instruments or implements intended by the owner pf the tenement for an industry ot works which may be carried on in a building or

on a piece of land, and which tend directly to meet the needs of the said industry or works; Appellant should have registered its protest before or at the time of the sale of the property. While not conclusive, the appellant's characterization of the property as chattels is indicative of intention and impresses upon the property the character determined by the parties. Machinery is naturally movable. However, machinery may be immobilized by destination or purpose under the following conditions: General Rule: The machinery only becomes immobilized if placed in a plant by the owner of the property or plant. Immobilization cannot be made by a tenant, a usufructuary, or any person having only a temporary right. Exception: The tenant, usufructuary, or temporary possessor acted as agent of the owner of the premises; or he intended to permanently give away the property in favor of the owner. As a rule, therefore, the machinery should be considered as Personal Property, since it was not placed on the land by the owner of the said land. MANILA ELECTRIC CO. V. CENTRAL BOARD OF ASSESSMENT APPEALS FACTS: Petitioner owns two oil storage tanks, made of steel plates wielded and assembled on the spot. Their bottoms rest on a foundation consisted of compacted earth, sand pad as immediate layer, and asphalt stratum as top layer. The tanks merely sit on its foundation. The municipal treasurer of Batangas made an assessment for realty tax on the two tanks, based on the report of the Board of Assessors. MERALCO wished to oppose this assessment as they averred that the tanks are not real properties.

HELD: While the two storage tanks are not embodied in the land, they may nevertheless be considered as improvements in the land, enhancing its utility and rendering it useful to the oil industry. For purposes of taxation, the term real property may include things, which should generally be considered as personal property. it is familiar phenomenon to see things classified as real property for purposes of taxation which on general principle may be considered as personal property. BURGOS Doctrine: A machinery which is movable by nature becomes immobilized when placed by the owner of the tenement, property or plant, but not so when placed by a tenant, usufructuary, or any other person having only temporary right, unless such person acted as the agent of the owner. Facts: Armed with a search warrant issued by the Court of First Instance of Rizal, law enforcement officers searched the offices of the We forum and Metropolitan Mail newspapers. During the course of the search, the law enforcement officers seized office and printing machines, equipment, paraphernalia and several other materials used in the distribution of newspapers. Petitioner avers, among others, that the seizure of the properties mentioned above amounts to seizure of real properties, which cannot be validly conducted under the strength of a search warrant. It must be noted that real properties are not susceptible of confiscation under a search warrant. Hence this appeal which assails the validity of the search and the seizure of the properties of the petitioner.

Issue: Whether there is merit in the petitioners assertion that real property were invalidly seized under the disputed warrants. Held: No. The petitioners assertion does not hold water. Under Article 415(5) of the civil code, machinery, receptacles, instruments or implements intended by the owner of the tenement for an industry or works which may be carried on in a building or on a piece of land and which tend directly to meet the needs of the said industry or works are considered immovable property. In another case decided by the Court, in which the abovementioned legal provision was invoked, it was ruled that machinery which is movable by nature becomes immobilized when placed by the owner of the tenement, property or plant, but not so when placed by a tenant, usufructuary, or any other person having only temporary right, unless such person acted as the agent of the owner. In the case at bar, petitioners do not claim to be the owners of the land and/or building on which the machineries were placed. This being the case, the machineries in question, while in fact bolted to the ground remains movable property susceptible to seizure under a search warrant. Luis Marcos Laurel vs Hon. Zeus Abrogar GR No. 155076 January 13, 2009 FACTS Laurel was charged with Theft under Art. 308 of the RPC for allegedly taking, stealing, and using PLDT's international long distance calls by conducting International Simple Resale (ISR) a method of outing and completing international long-distance calls using lines, cables, antennae, and/or air wave frequency which connect directly to the local/domestic exchange facilities of the country where the call is destined. PLDT alleged that this service was stolen from them using their own equipment and caused damage to them amounting to P20,370,651.92. PLDT alleges that the international calls and business of providing telecommunication or telephone service are personal properties capable of appropriation and can be objects of theft. ISSUE WON Laurel's act constitutes Theft HELD Art.308, RPC: Theft is committed by any person who, with intent to gain but without violence against, or intimidation of persons nor force upon things, shall take personal property of another without the latters consent. Elements of Theft under Art.308, RPC: 1. There be taking of Personal Property; 2. Said Personal Property belongs to another; 3. Taking be done with Intent to Gain; 4. Taking be done without the owners consent; 5. No violence against, or intimidation of, persons or force upon things Personal Property anything susceptible of appropriation and not included in Real Property Thus, the term personal property as used in Art.308, RPC should be interpreted in the context of the Civil Code's definition of real and personal property. Consequently, any personal property, tangible or intangible, corporeal or incorporeal, capable of appropriation may be the subject of theft (*US v Carlos; US v Tambunting; US v Genato*), so long as the same is not included in the enumeration of Real Properties under the Civil Code. The only requirement for personal property to capable of theft, is that it be subject to appropriation. Art. 416 (3) of the Civil Code deems Forces of Nature which are brought under the control of science, as Personal Property.

The appropriation of forces of nature which are brought under control by science can be achieved by tampering with any apparatus used for generating or measuring such forces of nature, wrongfully redirecting such forces of nature from such apparatus, or using any device to fraudulently obtain such forces of nature. In the instant case, the act of conducting ISR operations by illegally connecting various equipment or apparatus to PLDTs telephone system, through which petitioner is able to resell or re-route international long distance calls using PLDTs facilities constituteSubtraction. Moreover, interest in business should be classified as personal property since it is capable of appropriation, and not included in the enumeration of real properties. Therefore, the business of providing telecommunication or telephone service are personal property which can be the object of theft under Art. 308 of the RPC. The act of engaging in ISR is an act of subtraction penalized under the said article. While international long-distance calls take the form of electrical energy and may be considered as personal property, the said long-distance calls do not belong to PLDT since it could not have acquired ownership over such calls. PLDT merely encodes, augments, enhances, decodes and transmits said calls using its complex communications infrastructure and facilities. Since PLDT does not own the said telephone calls, then it could not validly claim that such telephone calls were taken without its consent. What constitutes Theft is the use of the PLDT's communications facilities without PLDT's consent. The theft lies in the unlawful taking of the telephone services & businesses. The Amended Information should be amended to show that the property subject of the theft were services and business of the offended party. MAKATI LEASING AND FINANCE CORP. V. WEAREVER TEXTILE MILLS, INC. Parties to a contract may by agreement treat as personal property that which by nature is a real property, as long as no interest of 3rd party would be prejudiced. FACTS: To obtain financial accommodations from Makati Leasing, Wearever Textile discounted and assigned several receivables under a Receivable Purchase Agreement with Makati Leasing. To secure the collection of receivables, it executed a chattel mortgage over several raw materials and a machinery Artos Aero Dryer Stentering Range (Dryer). Wearever defaulted thus the properties mortgaged were extrajudicially foreclosed. The sheriff, after the restraining order was lifted, was able to enter the premises of Wearever and removed the drive motor of the Dryer. The CA reversed the order of the CFI, ordering the return of the drive motor since it cannot be the subject of a replevin suit being an immovable bolted to the ground. Thus the case at bar. ISSUE: Whether the dryer is an immovable property HELD: NO The SC relied on its ruling in Tumalad v. Vicencio, that if a house of strong materials can be the subject of a Chattel Mortgage as long as the parties to the contract agree and no innocent 3rd party will be prejudiced then moreso that a machinery may treated as a movable since it is movable by nature and becomes

immobilized only by destination. And treating it as a chattel by way of a Chattel Mortgage, Wearever is estopped from claiming otherwise. Caltex vs Central Board of Assessment Appeals Posted on June 24, 2013 Caltex vs Central Board of Assessment Appeals & City Assessor of Pasay GR No. L-50466 May 31, 1982 This case is about the realty tax on machinery and equipment installed by Caltex (Philippines) Inc., in its gas stations located on leased land. FACTS Caltex loaned machines and equipment to gas station operators under an appropriate lease agreement or receipt. The lease contract stipulated that upon demand, the operators shall return to Caltex the machines and equipment in good condition as when received, ordinary wear and tear excepted. The lessor of the land, where the gas station is located, does not become the owner of the machines and equipment installed therein. Caltex retains the ownership thereof during the term of the lease. The City Assessor of Pasay City characterized the said items of gas station equipment and machinery as taxable realty. However, the City Board of Tax Appeals ruled that they are personalty. The Assessor appealed to the Central Board of Assessment Appeals. The Board held on June 3, 1977 that the said machines are real property within the meaning of Ses. 3(k) & (m) and 38 of the Real Property Tax Code, PD 464, and that the Civil Code definitions of real and personal property in Articles 415 and 416 are not applicable in this case. ISSUE WON the pieces of gas station equipment and machinery permanently affixed by Caltex to its gas station and pavement should be subject to realty tax. HELD Sec.2 of the Assessment Law provides that the realty tax is due on real property, including land, buildings, machinery, and other improvements not specifically exempted in Sec.3 thereof. Sec.3 of the Real Property Tax Code provides the following definitions: k) Improvements a valuable addition made to property or an amelioration in its conditionmore than mere repairs or replacement of wasteintended to enhance its value, beauty, or utility m) Machinery machines, mechanical contrivances, instruments, appliances, and apparatus attached to the real estateincludes the physical facilities available for productioninstallation and appurtenant service facilities. The subject machines and equipment are taxable improvement and machinery within the meaning of the Assessment Law and the Real Property Tax Code, because the same are necessary to the operation of the gas station and have been attached/affixed/embedded permanently to the gas station site. Improvements on land are commonly taxed as realty even though they might be considered personalty. It is a familiar phenomenon to see things classified as real property for purposes of taxation which on general principle might be considered personal property (Standard Oil Co., vs Jaramillo, 44 PHIL 630). This case is also easily distinguishable from Board of Assessment Appeals vs. Manila Electric Co., (119 Phil. 328) where Meralco's steel towers were exempted from taxation. The steel towers were considered personalty because they were attached to square metal frames by means of bolts and could be moved from place to place when unscrewed and dismantled. Nor are Caltex's gas station equipment and machinery the same as the tools and equipment in the repair shop of a bus company which were held to be personal

property not subject to realty tax (Mindanao Bus Co. vs. City Assessor, 116 Phil. 501). The Central Board of Assessment Appeals did not commit a grave abuse of discretion in upholding the City Assessor's imposition of the realty tax on Board of Assessment Appeals QC v MERALCO Posted on June 22, 2013 Board of Assessment Appeals, Q.C. vs Meralco 10 SCRA 68 GR No. L-15334 January 31, 1964 FACTS On November 15, 1955, the QC City Assessor declared the MERALCO's steel towers subject to real property tax. After the denial of MERALCO's petition to cancel these declarations, an appeal was taken to the QC Board of Assessment Appeals, which required respondent to pay P11,651.86 as real property tax on the said steel towers for the years 1952 to 1956. MERALCO paid the amount under protest, and filed a petition for review in the Court of Tax Appeals (CTA) which rendered a decision ordering the cancellation of the said tax declarations and the refunding to MERALCO by the QC City Treasurer of P11,651.86. ISSUE Are the steel towers or poles of the MERALCO considered real or personal properties? HELD Pole long, comparatively slender, usually cylindrical piece of wood, timber, object of metal or the like; an upright standard to the top of which something is affixed or by which something is supported. MERALCO's steel supports consists of a framework of 4 steel bars/strips which are bound by steel cross-arms atop of which are cross-arms supporting 5 highvoltage transmission wires, and their sole function is to support/carry such wires. The exemption granted to poles as quoted from Part II, Par.9 of respondent's franchise is determined by the use to which such poles are dedicated. It is evident that the word poles, as used in Act No. 484 and incorporated in the petitioner's franchise, should not be given a restrictive and narrow interpretation, as to defeat the very object for which the franchise was granted. The poles should be taken and understood as part of MERALCO's electric power system for the conveyance of electric current to its consumers. Art. 415 of the NCC classifies the following as immovable property: (1) Lands, buildings, roads and constructions of all kinds adhered to the soil; xxx (3) Everything attached to an immovable in a fixed manner, in such a way that it cannot be separated therefrom without breaking the material or deterioration of the object; xxx (5) Machinery, receptacles, instruments or implements intended by the owner pf the tenement for an industry ot works which may be carried on in a building or on a piece of land, and which tend directly to meet the needs of the said industry or works; Following these classifications, MERALCO's steel towers should be considered personal property. It should be noted that the steel towers: (a) are neither buildings or constructions adhered to the soil; (b) are not attached to an immovable in a fixed manner they can be separated without breaking the material or deterioration of the object; are not machineries, receptacles or instruments, and even if they are, they are not intended for an industry to be carried on in the premises.

BERKENKOTTER FACTS: This is an appeal taken by the plaintiff, B. H. Berkenkotter, from the judgment of the Court of First Instance of Manila, dismissing said plaintiffs complaint against Cu Unjieng e Hijos et al Mabalacat Sugar Co., Inc., owner of the sugar central situated in Mabalacat, Pampanga, obtained from the defendants, Cu Unjieng e Hijos, a loan secured by a first mortgage constituted on two parcels and land with all its buildings, improvements, sugar-cane mill, steel railway, telephone line, apparatus, utensils and whatever forms part or is necessary complement of said sugar-cane mill, steel railway, telephone line, now existing or that may in the future exist is said lots. Shortly after said mortgage had been constituted, the Mabalacat Sugar Co., Inc., decided to increase the capacity of its sugar central by buying additional machinery and equipment, so that instead of milling 150 tons daily, it could produce 250. The estimated cost of said additional machinery and equipment was approximately P100,000. B.A. Green, president of said corporation, proposed to the plaintiff, B.H. Berkenkotter, to advance the necessary amount for the purchase of said machinery and equipment, promising to reimburse him as soon as he could obtain an additional loan from the mortgagees, the herein defendants Cu Unjieng e Hijos. Berkenkotter agreed to the said proposition and delivered to him a total sum of P25,750. Berkenkotter had a credit of P22,000 against said corporation for unpaid salary. With the loan of P25,750 and said credit of P22,000, the Mabalacat Sugar Co., Inc., purchased the additional machinery and equipment now in litigation. B.A. Green, president of the Mabalacat Sugar Co., Inc., applied to Cu Unjieng e Hijos for an additional loan of P75,000 offering as security the additional machinery and equipment acquired by said B.A. Green and installed in the sugar central after the execution of the original mortgage deed, together with whatever additional equipment acquired with said loan. B.A. Green failed to obtain said loan. Appellants contention: the installation of the machinery and equipment claimed by him in the sugar central of the Mabalacat Sugar Company, Inc., was not permanent in character inasmuch as B. A. Green, in proposing to him to advance the money for the purchase thereof, that in case B. A. Green should fail to obtain an additional loan from the defendants Cu Unjieng e Hijos, said machinery and equipment would become security therefor. ISSUE: Whether or not the lower court erred in declaring that the additional machinery and equipment, as improvement incorporated with the central are subject to the mortgage deed executed in favor of the defendants Cu Unjieng e Hijos. HELD: No error was committed by trial court. The additional machinery and equipment are included in the first mortgage. Article 334, paragraph 5, of the Civil Code gives the character of real property to machinery, liquid containers, instruments or implements intended by the owner of any building or land for use in connection with any industry or trade being carried on therein and which are expressly adapted to meet the requirements of such trade or industry. If the installation of the machinery and equipment in question in the central of the Mabalacat Sugar Co., Inc., in lieu of the other of less capacity existing therein, for its sugar industry, converted them into real property by reason of their purpose, it cannot be said that their incorporation therewith was not permanent in character because, as essential and principal elements of a sugar central, without them the sugar central would be unable to function or carry on the industrial purpose for which it was established. Inasmuch as the central is permanent in character, the necessary machinery and equipment installed for

carrying on the sugar industry for which it has been established must necessary be permanent. Furthermore, the fact that B. A. Green bound himself to the plaintiff B. H. Berkenkotter to hold said machinery and equipment as security, as nothing could prevent B. A. Green from giving them as security at least under a second mortgage. As to the alleged sale of said machinery and equipment to the plaintiff and appellant after they had been permanently incorporated with the sugar central of the Mabalacat Sugar Co., Inc., and while the mortgage constituted on said sugar central to Cu Unjieng e Hijos remained in force, only the right of redemption of the vendor Mabalacat Sugar Co., Inc., in he sugar central with which said machinery and equipment had been incorporated, was transferred thereby, subject to the right of the defendants Cu Unjieng e Hijos under the first mortgage. For the foregoing considerations, we are of the opinion and so hold: (1) That the installation of a machinery and equipment in a mortgaged sugar central, in lieu of another of less capacity, for the purpose of carrying out the industrial functions of the latter and increasing production, constitutes a permanent improvement on said sugar central and subjects said machinery and equipment to the mortgage constituted thereon (article 1877, Civil Code); (2) that the fact that the purchaser of the new machinery and equipment has bound himself to the person supplying him the purchase money to hold them as security for the payment of the latters credit, and to refrain from mortgaging or otherwise encumbering them does not alter the permanent character of the incorporation of said machinery and equipment with the central; and (3) that the sale of the machinery and equipment in question by the purchaser who was supplied the money, after the incorporation thereof with the mortgaged sugar central, does not vest the creditor with ownership of said machinery and equipment but simply with the right of redemption. overnment v. Cabangis 53 Phil. 112 FACTS: In 1896, A owned a parcel of land, but because of the action of the waves of Manila Bay, part of said land was gradually submerged in the sea. It remained submerged until 1912 when the government decided to make the necessary dredging to reclaim the land from the sea. As soon as the land had been recovered A took possession of it. Issue: the ownership of the reclaimed land. HELD: The government owns the reclaimed land in the sense that it has become property of public dominion, because in letting it remain submerged, A may be said to have abandoned the same. Having become part of the sea or the seashore, it became property for public use. When the government took steps to make it land again, its status as public dominion remained unchanged; therefore, A is not entitled to the land. Cebu Oxygen and Acetylene Co., Inc. v. Bercilles L-40474, Aug. 29, 1975 FACTS: The City Council of Cebu, in 1968, considered as an abandoned road, the terminal portion of one of its streets. Later it authorized the sale thru public bidding of the property. The Cebu Oxygen and Acetylene Co. was able to purchase the same. It then petitioned the RTC of Cebu for the registration of the land. The petition was opposed by the Provincial Fiscal(Prosecutor) who argued that the lot is still part of the public domain, and cannot therefore be registered. Issue: May the lot be registered in the name of the buyer?

HELD: Yes, the land can be registered in the name of the buyer, because the street has already been withdrawn from public use, and accordingly has become patrimonial property. The lots sale was therefore valid. Salas V Jarencio (1972)Ponente: Esguerra, J.Legal Doctrine: Regardless of the source or classification of land in the possession of amunicipality, excepting those acquired with its own funds in its private or corporatecapacity, such property is held in trust for the State for the benefit of its inhabitants,whether it be for governmental or proprietary purposes Facts: February 24, 1919the 4th Branch of the Court of First Instance of Manila, acting asa land registration court, rendered judgment in Case No. 18, G.L.R.O. Record No. 111,declaring the City of Manila the owner in fee simple of a parcel of land known as LotNo. 1, Block 557 of the Cadastral Survey of the City of Mani1a, containing an area of 9,689.8 square meters, more or less. August 21, 1920 Title No. 4329 issued on in favor of the City of Manila after the landin question was registered in the City's favor. The Torrens Title expressly states thatthe City of Manila was the owner in 'fee simple' of the said land September 20, 1960the Municipal Board, presided by then Vice-Mayor AntonioVillegas, requested "His Excellency the President of the Philippines to consider thefeasibility of declaring the city property bounded by Florida, San Andres andNebraska Streets, under Transfer Certificate of Title Nos. 25545 and 25547,containing an area of 7,450 square meters, as patrimonial property of the City of Manila for the purpose of reselling these lots to the actual occupants thereof The said resolution of the Municipal Board of the City of Manila was officiallytransmitted to the President of the Philippines the following day, to which a copy wasfurnished to the Senate and House of Representatives of the Congress of thePhilippines. June 20, 1964RA 4118 was passed by the Senate and approved by the Presidentpursuant to the request. Such bill was enacted for social justice purposes, that theybe sold to their currently landless occupants. But due to reasons which do not appear in the record, the City of Manila made acomplete turn-about, for on December 20, 1966, Antonio J. Villegas, in his capacity asthe City Mayor of Manila and the City of Manila as a duly organized publiccorporation, brought an action for injunction and/or prohibition with preliminaryinjunction to restrain, prohibit and enjoin the herein appellants, particularly theGovernor of the Land Authority and the Register of Deeds of Manila, from furtherimplementing Republic Act No. 4118, and praying for the declaration of Republic ActNo. 4118 as unconstitutional. Issues and Decisions: 1. Is the property involved private or patrimonial property of the City of Manila? NO , it isthe property of the State. 2. Is Republic Act No. 4118 valid and not repugnant to the Constitution? YES , it is valid.

Ratio:1.Is the property involved private or patrimonial property of the City of Manila? NO, it is the property of the State. The rule is that when it comes to property of the municipality which it did not acquire in itsprivate or corporate capacity with its own funds, the legislature can transfer itsadministration and disposition to an agency of the National Government to be disposed of according to its discretion. The possession of a municipality, excepting those acquired with its own funds in its privateor corporate capacity, such property is held in trust for the State for the benefit of itsinhabitants, whether it be for governmental or proprietary purposes. The City of Manila, although declared by the Cadastral Court as owner in fee simple, has notshown by any shred of evidence in what manner it acquired said land as its private orpatrimonial property. The presumption is that such land came from the State upon thecreation of the municipality. That it has in its name a registered title is not questioned, but this title should be deemed tobe held in trust for the State as the land covered thereby was part of the territory of the Cityof Manila granted by the sovereign upon its creation Therefore, the land in question pertains to the State and the City of Manila merely acted astrustee for the benefit of the people therein for whom the State can legislate in the exerciseof its legitimate powers. 2.Is Republic Act No. 4118 valid and not repugnant to the Constitution? YES,it is valid. Consequently, the City of Manila was not deprived of anything it owns, either under the dueprocess clause or under the eminent domain provisions of the Constitution. If it failed to getfrom the Congress the concession it sought of having the land involved given to it as itspatrimonial property, the Courts possess no power to grant that relief. Republic Act No. 4118does not, therefore, suffer from any constitutional infirmity Province of Zamboanga Del Norte v. City of Zamboanga, et al. L-24440, Mar. 28, 1968 FACTS: After Zamboanga Province was divided into two (Zamboanga del Norte andZamboanga del Sur), Republic Act 3039 was passed providing that All buildings, properties, and assets belonging to the former province of Zamboanga and located within the City of Zamboanga are hereby transferred free of charge in favor of the City of Zamboanga. Suit was brought alleging that this grant without just compensation was unconstitutional because it deprived the province of property without due process. Included in the properties were the capital site and capitol building, certain school sites, hospital and leprosarium sites, and high school playgrounds. Issues: a) Are the properties mentioned, properties for public use or patrimonial? b) Should the city pay for said properties? HELD: a) If we follow the Civil Code classifi cation, only the high school playgrounds are for public use (in the sense that generally, they are available to the general public), and all the rest are PATRIMONIAL (since they are not devoted to public use but to public service; since they are not for public use, under Art. 424 of the Civil Code, they are patrimonial. [NOTE: For public use if ANYBODY can use; for public service if only AUTHORIZED persons can use.]. [NOTE: Had they been owned by the STATE, they would not have been patrimonial but would have been properties of public dominion for this would include public service, conformably with Art. 420, par. 2.]. BUT if we follow the law of Municipal Corporations (and not the Civil Code), as long as the purpose is for a public service (governmental service like public

education, public health, local administration), the property should be considered for PUBLIC USE. b) If the Civil Code classifi cation is used, since almost all the properties involved are patrimonial, the law would be unconstitutional since the province would be deprived of its own property without just compensation. If the law on Municipal Corporations would be followed, the properties would be of public dominion, and therefore NO COMPENSATION would be required. It is this law on Municipal Corporations that should be followed. Firstly, while the Civil Code may classify them as patrimonial, they should not be regarded as ordinary private property. They should fall under the control of the State, otherwise certain governmental activities would be impaired. Secondly, Art. 424, 2nd paragraph itself says without prejudice to the provisions (or PRINCIPLES) of special laws. The Public Estates Authority is the central implementing agency tasked to undertake reclamation projects nationwide. It took over the leasing and selling functions of the DENR insofar as reclaimed or about to be reclaimed foreshore lands are concerned. PEA sought the transfer to AMARI, a private corporation, of the ownership of 77.34 hectares of the Freedom Islands. PEA also sought to have 290.156 hectares of submerged areas of Manila Bay to AMARI. ISSUE: Whether or not the transfer is valid. HELD: No. To allow vast areas of reclaimed lands of the public domain to be transferred to PEA as private lands will sanction a gross violation of the constitutional ban on private corporations from acquiring any kind of alienable land of the public domain. The Supreme Court affirmed that the 157.84 hectares of reclaimed lands comprising the Freedom Islands, now covered by certificates of title in the name of PEA, are alienable lands of the public domain. The 592.15 hectares of submerged areas of Manila Bay remain inalienable natural resources of the public domain. Since the Amended JVA seeks to transfer to AMARI, a private corporation, ownership of 77.34 hectares of the Freedom Islands, such transfer is void for being contrary to Section 3, Article XII of the 1987 Constitution which prohibits private corporations from acquiring any kind of alienable land of the public domain. Furthermore, since the Amended JVA also seeks to transfer to AMARI ownership of 290.156 hectares of still submerged areas of Manila Bay, such transfer is void for being contrary to Section 2, Article XII of the 1987 Constitution which prohibits the alienation of natural resources other than agricultural lands of the public domain. HARTY v. MUNICIPALITY OF VICTORIA, 226 U.S. 12 (1912) 226 U.S. 12 MGR. JEREMIAH J. HARTY, Appt., and Plff. in Err. v. MUNICIPALITY OF VICTORIA. No. 13. Argued October 30, 1912. Decided November 11, 1912. Messrs. Frederic R. Coudert and Harry W. Van Dyke for appellant and plaintiff in error. Mr. Felix Frankfurter for appellee and defendant in error. Mr. Justice Holmes delivered the opinion of the court: This suit was brought by the Archbishop of Manila to recover a square in the municipality of Victoria. The church of the town and its parish house stand in this square, and they are admitted to be church property, but the land not occupied by them was declared by the supreme court to constitute the public square or

plaza [226 U.S. 12, 13] of the town, devoted to public uses. The plaintiff brought a writ of error and appealed. The appeal must be dismissed. Jover y Costas v. Philippine Islands, 221 U.S. 623, 635 , 55 S. L. ed. 884, 890, 31 Sup. Ct. Rep. 664; Carino v. Philippine Islands, 212 U.S. 449, 456 , 53 S. L. ed. 594, 595, 29 Sup. Ct. Rep. 334. The suit is like an ordinary action at law, and can be brought to this court only by writ of error, as was done in Santos v. Holy Roman Catholic & Apostolic Church,212 U.S. 463 , 53 L. ed. 599, 29 Sup. Ct. Rep. 338, and Ker & Co. v. Couden, 223 U.S. 268 , 56 L. ed. 432, 32 Sup. Ct. Rep. 284. There is a motion to dismiss the writ of error also, on the ground that the value of the real estate in controversy does not exceed $25,000. Affidavits to that effect are offered, and the order allowing the writ purports to do so on affidavits of the plaintiff and two others, 'notwithstanding the fact that, by admission of counsel for plaintiff, it appears that the value of the parcel of land for which judgment was rendered in favor of the defendant municipality, exclusive of the value of the adjoining parcel of land with the church and convent situated thereon, title to which is recognized to be in the plaintiff, and damages thereto resulting from the aforesaid judgment, does not exceed $25,000.' We doubt whether the affidavits do not imply the same admission, and whether the action should not be dismissed on that ground. The affidavit of the plaintiff puts the value of the land in controversy at over $25,000 on the manifestly untenable ground that the church edifices are deprived of free egress and ingress by the decision, and the others seemingly mean that the parcel of land with the church buildings included is worth $30,000, the buildings being valued at $25,000, leaving $5,000 for the land in dispute. But the result is the same if we go further. The evidence was contradictory, and although we were invited to consider it on the one side in the light of the relation of the church to the community, and on the other in that of the custom by which the plaza is of the essence of a [226 U.S. 12, 14] town, we can do neither. There is no question of law before us, for it hardly was argued, and could not be with any seriousness, that the supreme court was not authorized to review the evidence under 497 of the Philippine Code, or that this court can consider whether it was right in finding the preponderance of evidence to be on the defendant's side. Appeal and writ of error dismissed. Lunod et al vs. Meneses | Torres, J. G.R. No. 4223 | August 19, 1908 FACTS Plaintiifs (Appellees) Nicolas Lunod and 7 others are owners of farmlands on the upper estates near a lake (Calalaran). Defendant-Appellant Higno Meneses is the owner of a fishpond and a strip of land in Paraanan adjoining said lake on one side and a river on the other. Paraan is the only outlet of water to the river from the lands of Lunod et al during rainy season. In 1901 Meneses converted the land in Paraan to a fishpond and by means of a dam and a bamboo net prevented the free passage of water through Paraan causing flood and damage of plantations in the upper estates. Lunod et al filed a complaint alleging that there exists in favor of their rice fields a statutory easement for more than 20 years before 1901 and praying that Meneses be ordered to remove the obstructions that impede the passage of water through Paraanan. TC ruled in favor of the plaintiffs. ISSUE

WON Meneses can be permitted to obstruct the flow of waters through his lands. HELD NO. But Lunod et al cannot prevent the defendant from building works to prevent his lands against influx of waters. RATIO Where a statutory easement exists between adjoining estates, the owner of the lower lands must not construct any work that may impair or obstruct an easement which consists in receiving the waters which naturally, and without the intervention of man, descend from more elevated lands; neither shall the owner of the latter construct any work that may increase the easement. The Civil Code allows that every owner may enclose his property by means of walls, dikes, fences, or any other device, but his right is limited by the easement with which his estate is charged. Since the plaintiffs can not prevent the defendant from protecting his lands against the influx of salt water; but the defendant could never be permitted to obstruct the flow of the waters through his lands to the river during the heavy rains, when the high lands in Calalaran and the lake in said place are flooded, thereby impairing the right of the owners of the dominant estates; the court advised that it is perhaps useful and advantageous to all parties that Meneses be made to build a another dike in addition to the old dike between the lake of said place and the low lands in Paraanan, for the purpose of preventing the salt waters of the river flooding (at high tide) not only the lowlands in Paraanan but also the higher ones of Calalaran and its lake. GERMAN MANAGEMENT AND SERVICES V. COURT OF APPEALS FACTS: Spouses Jose issued a power of attorney in favor of petitioner for the development of their parcel of land into a subdivision. Private respondents were occupying the land and petitioner advised them to vacate but they refused. Thereafter, petitioner continued their development and construction. Respondents then filed a case for forcible entry. The trial court dismissed the complaint and this was reversed by the CA.

G.R. NO. 168732, JUNE 29, 2007 The NPC constructed underground tunnels on the property of the respondents without their knowledge and consent and without any expropriation proceeding. It contended that it constructed an easement on the property. Was there taking of the property considering that the owners were deprived of their beneficial use and enjoyment of the same, hence, entitled to just compensation? Yes. The manner in which the easement was created by the NPC, violated the due process rights of the owners as it was without notice and indemnity to them and did not go through proper expropriation proceedings. NPC could have, at any time, validly exercised the power of eminent domain to acquire the easement over the property as this power encompasses not only the taking or appropriation of title to and possession of the expropriated property but likewise covers even the imposition of a mere burden upon the owner of the condemned property. (Rep. v. PLDT, 136 Phil. 20 (1969)). Significantly, though, landowners cannot be deprived of their right over their land until expropriation proceedings are instituted in court. The court must then see to it that the taking is for pubic use, that there is payment of just compensation and that there is due process of law. In disregarding this procedure and failing to recognize the owners ownership of the sub-terrain portion, NPC took a risk and exposed itself to greater liability with the passage of time. It must be emphasized that the acquisition of the easement is not without expense. The underground tunnels imposed limitations on the owners use of the property for an indefinite period and deprived them of its ordinary use. The owners are clearly entitled to the payment of just compensation. Notwithstanding the fact that NPC only occupied the sub-terrain portion, it is liable to pay not merely an easement fee but rather the full compensation for the land. This is so because, the nature of the easement practically deprived the owners of its normal beneficial use. The owners, as the owners of the property thus expropriated, are entitled to a just compensation which should be neither more nor less, whenever it is possible to make the assessment, than the money equivalent of said property. (NPC v. Ibrahim, et al., G.R. No. 168732, June 29, 2007). VALUATION OF THE PROPERTY. NPC contended that if ever it is liable, it should be made to pay the value of the land from the time it constructed the tunnels. Is the contention correct? Why? No. To it to use the date it constructed the tunnels as the date of valuation would be grossly unfair. First, it did not enter the land under warrant or color of legal authority or with intent to expropriate the same. It did not notify the owners and wrongly assumed that it had the right to dig the tunnels under their property. Secondly, the improvements introduced in no way contributed to an increase in the value of the land. The valuation should be based at the time of the discovery of the construction of the underground tunnels. (NPC v. Ibrahim, et al., G.R. No. 168732, June 29, 2007). It is undisputed that there is a legal easement of right-of-way in favor of the Republic. Andayas transfer certificates of title contained the reservation that the lands covered thereby are subject to the provisions of the Land Registration Act and the Public Land Act. Section 112 of the Public Land Act provides that land granted by patent shall be subject to a right-of-way not exceeding 60 meters in width for public highways, irrigation ditches, aqueducts, and other similar works of the government or any public enterprise, free of charge, except only for the value of the improvements existing thereon that may be affected. In view of this,

the Court of Appeals declared that all Republic needs to do is to enforce such right without having to initiate expropriation proceedings and without having to pay any just compensation. Hence, the Republic may appropriate the 701 square meters necessary for the construction of the floodwalls without paying for it. Is the Republic liable for just compensation if in enforcing the legal easement of right-of-way on a property, the remaining area would be rendered unusable and uninhabitable? Yes, it is liable to pay consequential damages if in enforcing the legal easement on Andayas property, the remaining area would be rendered unusable and uninhabitable. Taking, in the exercise of the power of eminent domain, occurs not only when the government actually deprives or dispossess the property owner of his property or of its ordinary use, but also when there is a practical destruction or material impairment of the value of his property. Using this standard, there was undoubtedly a taking of the remaining area of Andayas property. True, no burden was imposed thereon and Andaya still retained title and possession of the property. But, the nature and the effect of the floodwalls would deprive Andaya of the normal use of the remaining areas. It would prevent ingress and egress to the property and turn it into a catch basin for the floodwaters coming from the Agusan River. For this reason, Andaya is entitled to payment of just compensation, which must be neither more nor less than the monetary equivalent of the land. One of the basic principles enshrined in our Constitution is that no person shall be deprived of his private property without due process of law; and in expropriation cases, an essential element of due process is that there must be just compensation whenever private property is taken for public use. Noteworthy, Section 9, Article III of our Constitution mandates that private property shall not be taken for public use without just compensation. Bachrach Motors v. Talisay-Silay Milling [G.R. No. 35223. September 17, 1931.] En Banc, Romualdez (J): 7 concurring Facts: On 22 December 1923, the Talisay-Silay Milling Co., Inc., was indebted to the PNB. To secure the payment of its debt, it succeeded in inducing its planters, among whom was Mariano Lacson Ledesma, to mortgage their land to the bank. And in order to compensate those planters for the risk they were running with their property under that mortgage, the aforesaid central, by a resolution passed on the same date, and amended on 23 March 1928, undertook to credit the owners of the plantation thus mortgaged every year with a sum equal to 2% of the debt secured according to the yearly balance, the payment of the bonus being made at once, or in part from time to time, as soon as the central became free of its obligations to the bank, and of those contracted by virtue of the contract of supervision, and had funds which might be so used, or as soon as it obtained from said bank authority to make such payment. <It seems Mariano Lacson Ledesma is indebted from Bachrach Motor; the circumstance of which is not found in the case facts.> Bachrach Motor Co., Inc. filed a complaint against the Talisay-Silay Milling Co., Inc., for the delivery of the amount of P13,850 or promissory notes or other instruments of credit for that sum payable on 30 June 1930, as bonus in favor of Mariano Lacson Ledesma. The complaint further prays that the sugar central be ordered to render an accounting of the amounts it owes Mariano Lacson Ledesma by way of bonus, dividends, or otherwise, and to pay Bachrach Motors

HELD: Notwithstanding petitioners claim that it was duly authorized by the owners to develop the subject property, private respondents as actual possessors, can commence a forcible entry case against petitioner because ownership is not in issue. Forcible entry is merely a quieting process, and never determines the actual title to an estate. Title is not involved. Although admittedly petitioner may validly claim ownership based on the muniment of title it presented, such evidence doesnt responsively address the issue of prior actual possession raised in a forcible entry case. It must be stated that regardless of the actual condition of title to the property, the party in a peaceable quiet possession shall not be turned out by a strong hand, violence or terror. Thus, a party who can prove prior possession can recover such possession even against the owner himself. Whatver may be the character of his prior possession, if he has in favor priority in time, he has security that entitles him to remain on the property until he has been lawfully ejected by a person having a better right by accion publiciana or accion reivindicatoria. NPC V. IBRAHIM, ET AL., - EMINENT DOMAIN

a sum sufficient to satisfy the judgment mentioned in the complaint, and that the sale made by said Mariano Lacson Ledesma be declared null and void. The PNB filed a third party claim alleging a preferential right to receive any amount which Mariano Lacson Ledesma might be entitled from Talisay-Silay Milling as bonus. TalisaySilay answered the complaint that Mariano Lacson Ledesmas credit (P7,500) belonged to Cesar Ledesma because he had purchase it. Cesar Ledesma claimed to be an owner by purchase in good faith. At the trial all the parties agreed to recognize and respect the sale made in favor of Cesar Ledesma of the P7,500 part of the credit in question, for which reason the trial court dismissed the complaint and cross-complaint against Cesar Ledesma authorizing the central to deliver to him the sum of P7,500. And upon conclusion of the hearing, the court held that the Bachrach Motor Co., Inc., had a preferred right to receive the amount of P11,076.02 which was Mariano Lacson Ledesmas bonus, and it ordered the central to deliver said sum to Bachrach Motors. PNB appealed. The Supreme Court affirmed the judgment appealed from, as it found no merit in the appeal;, without express finding as to costs. 1. Civil Fruits under Article 355 of the Civil Code Article 355 of the Civil Code considers three things as civil fruits: First, the rents of buildings; second, the proceeds from leases of lands; and, third, the income from perpetual or life annuities, or other similar sources of revenue. According to the context of the law, the phrase u otras analogas refers only to rents or income, for the adjectives otras and analogas agree with the noun rentas, as do also the other adjectives perpetuas and vitalicias. The civil fruits the Civil Code understands one of three and only three things, to wit: the rent of a building, the rent of land, and certain kinds of income. 2. Bonus not a civil fruit; not an income of the land The amount of the bonus, according to the resolution of the central granting it, is not based upon the value, importance or any other circumstance of the mortgaged property, but upon the total value of the debt thereby secured, according to the annual balance, which is something quite distinct from and independent of the property referred to. As the bonus is not obtained from the land, it is not civil fruits of that land. It is neither rent of buildings, proceeds from lease of lands, or income under Article 355 of the Civil Code. IGNACIO v HILARIO (1946; Moran) Facts: Elias Hilario and his wife Dionisia Dres filed a complaintagainst Damian, Francisco and Luis Ignacio concerning theownership of a parcel of land, partly rice-land and partly residential.After the trial of the case, the lower court under Judge Alfonso Felix,rendered judgment holding Hilario and Dres as the legal owners of the whole property but conceding to the Ignacios the ownership of the houses and granaries built by them on the residential portionwith the rights of a possessor in good faith, in accordance with article361 of the Civil Code.Subsequently, in a motion filed in the same CFI (now handled byrespondent Judge Hon. Felipe Natividad), Hilario and Dres prayed foran order of execution alleging that since they chose neither to paythe Ignacios for the buildings nor to sell to them the

residential lot,the Ignacios should be ordered to remove the structure at their ownexpense and to restore Hilario and Dres in the possession of said lot.After hearing, the motion was granted by Judge Natividad. Hence,the petition for certiorari was filed by the Ignacios praying for (a) arestraint and annulment of the order of execution issued by JudgeNatividad; (b) an order to compel Hilario and Dres to pay them thesum of P2,000 for the buildings, or sell to them the residential lot forP45; or (c) a rehearing of the case for a determination of the rights of the parties upon failure of extra-judicial settlement. The Supreme Court set aside the writ of execution issued by JudgeNatividad and ordered the lower court to hold a hearing in theprincipal case wherein it must determine the prices of the buildingsand of the residential lot where they are erected, as well as theperiod of time within which Hilario and Dres may exercise theiroption either to pay for the buildings or to sell their land, and, in the

last instance, the period of time within which the Ignacios may payfor the land, all these periods to be counted from the date the judgment becomes executory or unappealable. After such hearing,the court shall render a final judgment according to the evidencepresented by the parties; with costs against Hilarion and Dres. 1. Right of retention of builder in good faith The owner of the building erected in good faith on a land owned byanother, is entitled to retain the possession of the land until he ispaid the value of his building, under article 453. Article 453 providesthat Necessary expenses shall be refunded to every possessor; butonly the possessor in good faith may retain the thing until suchexpenses are made good to him. Useful expenses shall be refundedto the possessor in good faith with the same right of retention, theperson who has defeated him in the possession having the option of refunding the amount of the expenses or paying the increase invalue which the thing may have acquired in consequence thereof." 2. Option of the landowner to pay for the building or sell hisland to the owner of the building; Right of remotion onlyavailable if he chose the latter and the owner of the buildingcannot pay The owner of the land, upon the other hand, has the option, underarticle 361, either to pay for the building or to sell his land to theowner of the building. Article 361 provides that The owner of landon which anything has been built, sown or planted in good faith, shallhave the right to appropriate as his own the work, sowing orplanting, after the payment of the indemnity stated in articles 453and 454, or to oblige the one who built or planted to pay the price of the land, and the one who sowed, the proper rent. He cannothowever refuse both to pay for the building and to sell the land andcompel the owner of the building to remove it from the land where itis erected. He is entitled to such remotion only when, after havingchosen to sell his land, the other party fails to pay for the same. 3. Order amends judgment substantially and thus null andvoid The order of Judge Natividad compelling the Ignacios to remove theirbuildings from the land belonging to Hilario and Dres only becausethe latter chose neither to pay for such buildings nor to sell the land,is null and void, for it amends substantially the judgment sought tobe executed and is, furthermore, offensive to articles 361 and 453 of the Civil Code. 4. Original decision did not become final as it failed todetermine the value of the buildings and of the lot; and thetime to which the option may be exercised In the decision of Judge Felix, the rights of both parties were welldefined under articles 361 and 453 of the Civil Code, but it failed todetermine the value of the buildings and of the lot where they areerected as well as the periods of time

within which the option maybe exercised and payment should be made, these particulars havingbeen left for determination apparently after the judgment hasbecome final. The procedure is erroneous, for after the judgment hasbecome final, no additions can be made thereto and nothing can bedone therewith except its execution. And execution cannot be had,the sheriff being ignorant as to how, for how much, and within whattime may the option be exercised, and certainty no authority isvested in him to settle these matters which involve exercise of judicial discretion. Thus, the judgment rendered by Judge Felix hasnever become final, it having left matters to be settled for itscompletion in a subsequent proceeding, matters which remainedunsettled up to the time the petition is filed in the present case.Francisco Depra vs. Agustin Dumlao GR L57348 16 May 1985Facts: Depra is the owner of a parcel of land to which Dumlao, livingin an adjoining lot, had built a kitchen that encroached an area of 34square meters. The encroachment was discovered in a relocationsurvey of Depras property. Upon discovery, Depras mother wrote ademand letter asking Dumlao to move back from his encroachment.She then filed a case of Unlawful Detainer against Dumlao.In the trial court it was proven that Dumlao was a builder in goodfaith; thus the Municipal Court rendered it judgment that reads:Ordering that a forced lease is created between the parties with theplaintiffs, as lessors, and the defendants as lessees, over thedisputed portion with an area of thirty four (34) square meters, therent to be paid is five (P5.00) pesos a month, payable by the lesseeto the lessors within the first five (5) days of the month the rent isdue; and the lease shall commence on the day that this decisionshall have become final.Neither party appealed. However, Depra did not accept the paymentof rentals so that Dumlao deposited such rentals with the MunicipalCourt.Depra then filed a Complaint for Quieting of Title against Dumlao,the latter admitted the encroachment but alleged, that the present suit us barred by res judicate by virtue of the decision of theMunicipal Court.DEPRA claims that the Decision of the Municipal Court was null andvoid ab initio because its jurisdiction is limited to the sole issue of possession, whereas decisions affecting lease, which is anencumbrance on real property, may only be rendered by Courts of First Instance.Issue:I. Whether or not the Municipal Courts decision was null and void abinitio because it has no jurisdiction over the case?II. Whether or not the factual situations of DUMLAO and DEPRAconform to the juridical positions respectively defined by law, for a"builder in good faith" under Article 448, a "possessor in good faith"under Article 526 and a "landowner in good faith' under Article 448?Held:I. Addressing out selves to the issue of validity of theDecision of theMunicipal Court, we hold the same to be null and void. The judgmentin a detainer case is effective in respect of possession only (Sec. 7,Rule 70, Rules of Court). The Municipal Court over-stepped itsbounds when it imposed upon the parties a situation of "forcedlease", which like "forced co-ownership" is not favored in law.Furthermore, a lease is an interest in real property, jurisdiction overwhich belongs to Courts of First Instance (now Regional Trial Courts)(Sec. 44(b), Judiciary Act of 1948; Sec. 19 (2) Batas Pambansa Blg.129). Since the Municipal Court, acted without jurisdiction, itsDecision was null and void and cannot operate as res judicata to thesubject complaint for Queting of Title. Besides, even if the Decisionwere valid, the rule on res judicata would not apply due to differencein cause of action. In the Municipal Court, the cause of action was thedeprivation of possession, while in the action to quiet title, the causeof action was based on ownership. Furthermore, Sec. 7, Rule 70 of the Rules of Court explicitly provides that judgment in a detainercase "shall not bar an action between the same parties respectingtitle to the land. "II. ART. 448. The owner of the land on which anything has been builtsown or planted in good faith,shall have the rightto

appropriate as his own the works, sowing or planting, afterpayment of the indemnity provided for in articles 546 and 548, orto oblige the one who built or planted to pay the price of the land,and the one who sowed, the proper rent.However, the builder or planter cannot be obliged to buy the land if its value is considerably more than that of the building or trees. Insuch case, he shall pay reasonable rent, if the owner of the land doesnot choose to appropriate the building or trees after properindemnity. The parties shall agree upon the terms of the lease and incase of disagreement, the court shall fix the terms thereof (Paragraphing supplied)Pursuant to the foregoing provision, DEPRA has the option either topay for the encroaching part of DUMLAO's kitchen, or to sell theencroached 34 square meters of his lot to DUMLAO. He cannot refuseto pay for the encroaching part of the building, and to sell theencroached part of his land, 5 as he had manifested before theMunicipal Court. But that manifestation is not binding because it wasmade in a void proceeding.However, the good faith of DUMLAO is part of the Stipulation of Factsin the Court of First Instance. It was thus error for the Trial Court tohave ruled that DEPRA is "entitled to possession," without more, of the disputed portion implying thereby that he is entitled to have thekitchen removed. He is entitled to such removal only when, afterhaving chosen to sell his encroached land, DUMLAO fails to pay forthe same. 6 In this case, DUMLAO had expressed his willingness topay for the land, but DEPRA refused to sell. SARMIENTO V. AGANA129 SCRA 122 FACTS: While Ernesto Valentino was still courting his wife, latters motheroffered a lot for the construction of house by the spouses. Itwas assumed that the wifes mother was the owner of the land,which would eventually transfer to the spouses. It turned out thatSarmiento was the owner of the land. Sarmiento filed an ejectmentsuit to which the trial court found out that the spouses arepossessors in good faith and ordered Sarmiento to exercise optionbased on Art 448. Sarmiento did not exercise any of the options. The spouses then consigned the amount in court.ISSUE: Whether or not Sarmiento can refuse to exercise the givenoptions HELD: Negative. The landowner cannot refuse both toappropriate or sell the land, and to compel the builder to removeit from the land on which it is located. He is entitled to suchdemolition only when after having chosen to sell the land, the otherparty fails to pay for the same LEONOR GRANA and JULIETA TORRALBA VS. THE COURT OF APPEALS, AURORA BONGATO and JARDENIO SANCHEZ GR L-12486 31 AUG 1960 Facts: The herein Petitioners were sued by Bongato and Sanchez for the recovery of 87 square meters of residential land which they have inherited as the children of the spouses Marcos Bongato and Eusebia. The former were ordered by the to vacate and deliver it to said respondents and to pay a monthly rental of P10.00 from the filing of the complaint until they actually vacate the same, plus attorney's fees and costs. The Petitioners alleged that the said property became a subject of a cadastral survey due to conflicts and overlapping of boundaries. In that survey, Gregorio Bongato's lot, according to petitioners, was identified as Lot No. 311 and that of Isidaria Trillo, their predecessor in interest, as Lot No. 310. Citing the fact that Original Certificate of Title No. RO-72 (138) covers 295 square meters of land, while the sketch plan of the second cadastral survey of Butuan shows that Lot

No. 311 has only 230 square meters, petitioners maintain that it is the latter area properly belongs to respondents and that the land in question is part of the adjoining land, Lot No. 310, which belonged to their predecessor in interest. Issue: Whether or not the first survey was erroneous or that it included part of the contiguous land of petitioners' predecessor in interest? Held: Petitioners' stand is untenable. No proof was presented to show that the first survey was erroneous or that it included part of the contigous land of petitioners' predecessor in interest as part of the lot now covered by Original Certificate of Title No. RO-72 (138). Note that the difference in area between the land covered by said title and Lot No. 311 of the resurvey plan is 65 square meters while the area of the land in dispute if 87 square meters. And what is more, the alleged sketch plan of the resurvey was not presented in evidence. Upon the other hand, it is not disputed that the land in question is part of the lot covered by the Torrens title issued way back in 1923 in the name of respondents' predecessor in interest. Said title has not been contested up to the present, and, therefore, has become inconvertible evidence of the ownership of the land covered by it. Well settled is the rule that a Torrens certificate of title becomes conclusive and indefeasible after the lapse of the period within which it may be impugned (Reyes, et al. vs. Borbon, et al., 50 Phil., 791; Yumul vs. Rivera, et al., 64 Phil., 13). Although without any legal and valid claim over the land in question, petitioners, however, were found by the Court of Appeals to have constructed a portion of their house thereon in good faith. Under Article 361 of the old Civil Code (Article 448 of the new), the owner of the land on which anything has been built in good faith shall have the right to appropriate as his own faith shall have the right to appropriate as his own the building, after payment to the builder of necessary and useful expenses, and in the proper case, expenses for pure luxury or mere pleasure, or to oblige the builder to pay the price of the land. Respondents, as owners of the land, have therefore the choice of either appropriating the portion of petitioners' house which is on their land upon payment of the proper indemnity to petitioners, or selling to petitioners that part of their land on which stands the improvement. It may here be pointed out that it would be impractical for respondents to choose to exercise the first alternative, i.e., buy that portion of the house standing on their land, for in that event the whole building might be rendered useless. The more workable solution, it would seem, is for respondents to sell to petitioners that part of their land on which was constructed a portion of the latter's house. If petitioners are unwilling or unable to buy, then they must vacate the land and must pay rentals until they do so. Of course, respondents cannot oblige petitioners to buy the land if its value is considerably more than that of the aforementioned portion of the house. If such be the case, then petitioners must pay reasonable rent. The parties must come to an agreement as to the conditions of the lease, and should they fail to do so, then the court shall fix the same. (Article 361, old Civil Code; Article 448 of the new). DEPRA V. DUMLAO 136 SCRA 475 FACTS: Francisco Depra, is the owner of a parcel of land registered, situated in the municipality of Dumangas, Iloilo. Agustin Dumlao, defendant-appellant, owns an adjoining lot. When DUMLAO constructed his house on his lot, the kitchen thereof had encroached on an area of thirty four (34) square meters of DEPRAs property, After the encroachment was discovered in a relocation survey of

DEPRAs lot made on November 2,1972, his mother, Beatriz Depra after writing a demand letter asking DUMLAO to move back from his encroachment, filed an action for Unlawful Detainer. Said complaint was later amended to include DEPRA as a party plaintiff. After trial, the Municipal Court found that DUMLAO was a builder in good faith, and applying Article 448 of the Civil Code. DEPRA did not accept payment of rentals so that DUMLAO deposited such rentals with the Municipal Court. In this case, the Municipal Court, acted without jurisdiction, its Decision was null and void and cannot operate as res judicata to the subject complaint for Queting of Title. The court conceded in the MCs decision that Dumlao is a builder in good faith. Held: Owner of the land on which improvement was built by another in good faith is entitled to removal of improvement only after landowner has opted to sell the land and the builder refused to pay for the same. Res judicata doesnt apply wherein the first case was for ejectment and the other was for quieting of title. ART. 448. The owner of the land on which anything has been built sown or planted in good faith, shall have the right to appropriate as his own the works, sowing or planting, after payment of the indemnity provided for in articles 546 and 548, or to oblige the one who built or planted to pay the price of the land, and the one who sowed, the proper rent. However, the builder or planter cannot be obliged to buy the land if its value is considerably more than that of the building or trees. In such case, he shall pay reasonable rent, if the owner of the land does not choose to appropriate the building or trees after proper indemnity. The parties shall agree upon the terms of the lease and in case of disagreement, the court shall fix the terms thereof. HEIRS OF NAVARRO V. IAC Accretion along an area adjacent to the sea is public domain, even if the accretion results from rivers emptying into the sea. It cannot be registered. FACTS: Sinforoso Pascual sits in the midst of a land registration case. The story begins on 1946 upon his desire to register land on the northern section of his existing property. His current registered property is bounded on the east by Talisay River, on the West by Bulacan River and on the North by the Manila bay. Both rivers flow towards the Manila Bay. Because of constantly flowing water, extra land of about 17hectares (thats about the size of Disney Park!) formed in the northern most section of the property. It is this property he sought to register. The RTC denied the registration claiming this to be foreshore land and part of public domain (remember, accretion formedby the sea is public dominion). His Motion for Reconsideration likewise burned. In 1960, he attempted registry again, claiming that the Talisay and Bulacan rivers deposited more silt resulting on accretion. He claimed this land as riprarian owner. The Director of Lands, Director of Forestry and the Fiscal opposed. Then a new party surfaced. Mr Emiliano Navarro jumped into the fray opposing the same application, stating the he leased part of the property sought to be registered. He sought to protect his fishpond that rested on the same property. Sinforoso was not amused and filed ejectment against Mr. Navarro, claiming that Navarro used stealth force and strategy to occupy a portion of his land. Pascual

lost the case against Navarro so he appealed. During the appeal, his original land registration case was consolidated and tried jointly. (alas Pascual died) The heirs of Pascual took over the case. On 1975, the court decided that the property was foreshore land and therefore part of public domain. The RTC dismissed the complaint of Pascual for ejectment against Navarro and also denied his land registration request. Pascuals heirs appealed and the RTC was reversed by the IAC. The Apellate court granted petition for registration! The reason? The accretion was caused by the two rivers, not manila bay. Hence it wasnt foreshore land. (BUT the confusion lies in the fact that the accretion formed adjacent to Manila Bay which is sea!) Aggrieved, the Director of Forestry moved for reconsideration (Government insists it is foreshore and hence, public domain). The Apellate court denied all motions of the Director and the Government. The matter went to the SC. ISSUE: Whether or not the accretion taking place on property adjacent to the sea can be registered under the Torrens system. HELD: It cannot be registered. This is land of Public domain. Pascual claimed ownership under Article 457 of the Civil Code saying that the disputed 14-hectare land is an accretion caused by the joint action of the Talisay and Bulacan Rivers Art 457: Accretion as a mode of acquiring property and requires the concurrence of the following requisites: (1) that the accumulation of soil or sediment be gradual and imperceptible; (2) that it be the result of the action of the waters of the river; and (3) that the land where the accretion takes place is adjacent to the bank of the river. Unfortunately, Pasucal and Heirs claim of ownership based on Art 457 is misplaced. If theres any land to be claimed, it should be land ADJACENT to the rivers Talisay and Bulacan. The law is clear on this. Accretion of land along the river bank may be registered. This is not the case of accretion of land on the property adjacent to Manila Bay. Furthermore, Manila Bay is a sea. Accretion on a sea bank is foreshore land and the applicable law is not Art 457 but Art 4 of the Spanish Law of Waters of 1866. This law, while old, holds that accretion along sea shore cannot be registered as it remains public domain unless abandoned by government for public use and declared as private property capable of alienation. Article 4 of the Spanish Law of Waters of August 3, 1866 provides as follows: Lands added to the shores by accretions and alluvial deposits caused by the action of the sea, form part of the public domain. When they are no longer washed by the waters of the sea and are not necessary for purposes of public utility, or for the establishment of special industries, or for the coast-guard service, the Government shall declare them to be the property of the owners of the estates adjacent thereto and as increment thereof. The IAC decision granting registration was reversed and set aside. Registration cannot be allowed. REPUBLIC V. CA 132 SCRA 514 FACTS:

Respondents sought the registration of land adjacent to their fishpond. They are the registered owners of parcel of lot bordering on the Bocaue and Meycauyan rivers. The lower and appellate court allowed registration but this was opposed by the government.

HELD: There is no accretion if by man-made causes. REP. OF THE PHIL. VS. CA, ET. AL.: Case Doctrine: The requirement that the deposit should be due to the effect of the current of the river isindispensable. This excludes from Art. 457 of the New Civil Code all deposits caused by human intervention.Alluvion must be the exclusive work of nature. FACTS: Respondents Benjamin Tancinco, Azucena Tancinco Reyes, Marina (should be "Maria") Tancinco Imperialand Mario C. Tancinco filed an application for the registration of three lots adjacent to their fishpond property Assistant Provincial Fiscal Amando C. Vicente, in representation of the Bureau of Lands filed a written opposition tothe application for registration.Petitioner submits that there is no accretion to speak of under Article 457 of the New Civil Code because whatactually happened is that the private respondents simply transferred their dikes further down the river bed of theMeycauayan River, and thus, if there is any accretion to speak of, it is man-made and artificial and not the result of the gradual and imperceptible sedimentation by the waters of the river.On the other hand, private respondents submit the fact of accretion without human intervention because thetransfer of the dike occurred after the accretion was complete. ISSUE/S: WON there was accretion HELD: There is no accretion.Article 457 of the NCC, requires the concurrence of three requisites before an accretion covered by this particularprovision is said to have taken place. They are (1) that the deposit be gradual and imperceptible; (2) that it bemade through the effects of the current of the water; and (3) that the land where accretion takes place is adjacentto the banks of rivers.The requirement that the deposit should be due to the effect of the current of the river is indispensable. Thisexcludes from Art. 457 of the New Civil Code all deposits caused by human intervention. Alluvion must be theexclusive work of nature. In the instant case, there is no evidence whatsoever to prove that the addition to the saidproperty was made gradually through the effects of the current of the Meycauayan and Bocaue rivers. We agreewith the observation of the Solicitor General that it is preposterous to believe that almost four (4) hectares of land came into being because of the effects of the Meycauayan and Bocaue rivers.xxxxxxxxx However, there is evidence that the alleged alluvial deposits were artificial and man-made and not the exclusive result of the currentof the Meycauayan and Bocaue rivers. The alleged alluvial deposits came into being not because of the sole effectof the current of the rivers but as a result of the transfer of the dike towards the river and encroaching upon it. Theland sought to be registered is not even dry land cast imperceptibly and gradually by the river's current on thefishpond adjoining it. It is under two meters of water. The private respondents' own evidence shows that the waterin the fishpond is two meters deep on the side of the pilapil facing the fishpond and only one meter deep on theside of the pilapil facing the riverThe reason behind the law giving the riparian owner the right to any land or alluvion deposited by a river is tocompensate him for the danger of loss that he

suffers because of the location of his land. If estates bordering onrivers are exposed to floods and other evils produced by the destructive force of the waters and if by virtue of lawful provisions, said estates are subject to incumbrances and various kinds of easements, it is proper that therisk or danger which may prejudice the owners thereof should be compensated by the right of accretion. (Cortes v.City of Manila, 10 Phil. 567). Hence, the riparian owner does not acquire the additions to his land caused by specialworks expressly intended or designed to bring about accretion. When the private respondents transferred theirdikes towards the river bed, the dikes were meant for reclamation purposes and not to protect their property fromthe destructive force of the waters of the river LUCASAN V. PDIC (CIVIL, REDEMPTION, PROPERTY) To avail of the remedy of QUIETING OF TITLE, two indispensable requisites must concur, namely: the plaintiff or complainant has a legal or an equitable title to or interest in the real property subject of action; and the deed, claim, encumbrance or proceeding claimed to be casting cloud on his title must be shown in fact invalid or inoperative despite it prima facie appearance of validity or legal efficacy. Stated simply, the plaintiff must show that he has a legal title or at least an equitable title over the real property in dispute, and that some deed or proceeding beclouds its validity or efficacy.

Unfortunately, the foregoing requisites are wanting in this case. Lucasan admitted that he failed to redeem the property during the redemption period, on account of his ten limited financial situation. It was only 15 years later that he manifested his desire to reacquire the properties. Clearly thus, he had lost whatever right ha had over the lots. The payment of loans made by Lucasan cannot in any way operate to restore whatever right he had over the subject properties. Such payment only extinguished his loan obligations to the mortgagee banks and the liens which Lucasan claimed were subsisting at the time of the registration of the notice of embargo and certification of sale. Neither can Lucasan capitalize on PBC's failure to file a petition for consolidation of ownership after the expiration of the redemption period. with the rule that the expiration of the 1-year redemption period forecloses the obligor's right to redeem ans that the sale thereby becomes absolute, the issuance thereafter of a final deed of sale is at best a mere formality and mere confirmation of the title that is already vested in the purchaser. Certainly, Lucasan no longer possess any legal or equitable title to or interest over the subject parcels of land; hence, he cannot validly maintain an action for quieting of title. MANUEL T. DE GUIA, petitioner, vs. COURT OF APPEALS (Former Sixth Division) and JOSE B. ABEJO, representedby his Attorney-in-Fact, Hermenegilda AbejoRivera, respondents.2003 Oct 8 1st Division G.R. No. 120864 CASE DOCTRINES

Co-owner may file an action against a co-owner; purpose Any co-owner may file an action under Article 487 not only against a third person, but also againstanother co-owner who takes exclusive possession and asserts exclusive ownership of the property. In the lattercase, however, the only purpose of the action is to obtain recognition of the co-ownership. The plaintiff cannotseek exclusion of the defendant from the property because as co-owner he has a right of possession. The plaintiff cannot recover any material or determinate part of the property. Co-ownership; right of enjoyment The right of enjoyment by each co-owner is limited by a similar right of the other co-owners. A co-owner cannot devote common property to his exclusive use to the prejudice of the co-ownership. Hence, if thesubject is a residential house, all the co-owners may live there with their respective families to the extentpossible. However, if one co-owner alone occupies the entire house without opposition from the other co-owners, and there is no lease agreement, the other co-owners cannot demand the payment of rent. Conversely,if there is an agreement to lease the house, the co-owners can demand rent from the co-owner who dwells inthe house. FACTS:Petition for Review on Certiorari.Abejo instituted an action for recovery of possession with damages against DEGUIA. Abejos contentions: 1. he is the owner of the undivided portion of a property used as a fishpond registered Register of Deedsof Bulacan.2. ownership over approximately 39,611 square meters out of the FISHPONDs total area of 79,220 square meters Notes (Case Digests Property) DE GUIA continues to possess and use the FISHPOND without any contract and without paying rent to ABEJOs damage and prejudice. 4. DE GUIA refuses to surrender ownership and possession of the FISHPOND despite repeated demands to do so after DE GUIAs sublease contract over the FISHPOND had expired. 5. asked the trial court to order DE GUIA to vacate an approximate area of 39,611 square meters as well aspay damages.In his Answer, DE GUIA alleged:1. the complaint does not state a cause of action and has prescribed.2. the FISHPOND was originally owned by Maxima Termulo who died intestate with Primitiva Lejano as heronly heir.3. ABEJO is not the owner of the entire FISHPOND but the heirs of Primitiva Lejano who authorized him topossess the entire FISHPOND.4. ABEJOs ownership of the undivided portion of the FISHPOND as void and clai

med ownership over anundivided half portion of the FISHPOND for himself.5. DE GUIA sought payment of damages and reimbursement for the improvements he introduced as abuilder in good faith.RTC decision: in favor of Abejo.CA decision: affirmed the RTC.Issue 1: WON a co-owner may file an action for ejectment against a co-owner.Ruling: Article 487 of the Civil Code provides, *a+ny one of the co owners may bring an action in ejectment. This article covers all kinds of actions for the recovery of possession. Article 487 includes forcible entry and unlawfuldetainer (accion interdictal), recovery of possession (accion publiciana), and recovery of ownership (accion dereivindicacion). The summary actions of forcible entry and unlawful detainer seek the recovery of physicalpossession only. These actions are brought before municipal trial courts within one year from dispossession.However, accion publiciana, which is a plenary action for recovery of the right to possess, falls under the jurisdiction of the proper regional trial court when the dispossession has lasted for more than one year. Accion dereivindicacion, which seeks the recovery of ownership, also falls under the jurisdiction of the proper regional trialcourt.Any co-owner may file an action under Article 487 not only against a third person, but also againstanother co-owner who takes exclusive possession and asserts exclusive ownership of the property. In the lattercase, however, the only purpose of the action is to obtain recognition of the co-ownership. The plaintiff cannotseek exclusion of the defendant from the property because as co-owner he has a right of possession. The plaintiff cannot recover any material or determinate part of the property.In Hermogena G. Engreso with Spouse Jose Engreso v. Nestoria De La Cruz and Herminio De La Cruz, wereiterated the rule that a co-owner cannot recover a material or determinate part of a common property prior topartition as follows:It is a basic principle in civil law that before a property owned in common is actually partitioned, all thatthe co-owner has is an ideal or abstract quota or proportionate share in the entire property. A co-owner has noright to demand a concrete, specific or determinate part of the thing owned in common because until division iseffected his right over the thing is represented only by an ideal portion.As such, the only effect of an action brought by a co-owner against a co-owner will be to obtainrecognition of the co-ownership; the defendant cannot be excluded from a specific portion of the propertybecause as a co-owner he has a right to possess and the plaintiff cannot recover any material or determinatepart of the property. Thus, the courts a quo erred when they ordered the delivery of one-half () of the buildingin favor of private respondent.xxxxFollowing the inherent and peculiar features of co-ownership, while ABEJO and DE GUIA have equalshares in the FISHPOND quantitatively speaking, they have the same right in a qualitative sense as co-owners.Simply stated, ABEJO and DE GUIA are owners of the whole and over the whole, they exercise the right of dominion. However, they are at the same time individual owners of a portion, which is truly abstract becauseuntil there is partition, such portion remains indeterminate or unidentified. As co-owners, ABEJO and DE GUIA may jointly exercise the right of dominion over the entire FISHPOND until they partition the FISHPOND by identifying orsegregating their respective portions. ince a co-ownership subsists between ABEJO and DE GUIA, judicial or extrajudicial partition is theproper recourse. An action to demand partition is imprescriptible and not subject to laches. Each co-owner maydemand at any time the partition of the common property unless a co-owner has repudiated the co-ownershipunder certain conditions. Neither ABEJO nor DE GUIA has repudiated the co-ownership under the conditions set bylaw.To recapitulate, we

rule that a co-owner may file an action for recovery of possession against a coowner who takes exclusive possession of the entire co-owned property. However, the only effect of such actionis a recognition of the co-ownership. The courts cannot proceed with the actual partitioning of the co-ownedproperty. Thus, judicial or extra-judicial partition is necessary to effect physical division of the FISHPOND betweenABEJO and DE GUIA. An action for partition is also the proper forum for accounting the profits received by DE GUIAfrom the FISHPOND. However, as a necessary consequence of such recognition, ABEJO shall exercise an equalright to possess, use and enjoy the entire FISHPOND.Issue 2: WON it is proper for a co-owner to pay for rents while using the property.Ruling: The right of enjoyment by each co-owner is limited by a similar right of the other co-owners. A co-owner cannot devote common property to his exclusive use to the prejudice of the co-ownership. Hence, if thesubject is a residential house, all the co-owners may live there with their respective families to the extentpossible. However, if one co-owner alone occupies the entire house without opposition from the other co-owners, and there is no lease agreement, the other co-owners cannot demand the payment of rent. Conversely,if there is an agreement to lease the house, the co-owners can demand rent from the co-owner who dwells inthe house. The co-owners can either exercise an equal right to live in the house, or agree to lease it. If they fail toexercise any of these options, they must bear the consequences. It would be unjust to require the co-owner topay rent after the co-owners by their silence have allowed him to use the property.In case the coowners agree to lease a building owned in common, a co-owner cannot retain it for his usewithout paying the proper rent. Moreover, where part of the property is occupied exclusively by some co-ownersfor the exploitation of an industry, the other co-owners become co-participants in the accessions of the propertyand should share in its net profits. /adsum RICARDO PARDELL Y CRUZ and VICENTA ORTIZ Y FELIN DE PARDELL, plaintiffsappellees, vs. GASPAR DE BARTOLOME Y ESCRIBANO and MATILDE ORTIZ Y FELIN DE BARTOLOME, defendants-appellants. 1912 Nov 18 1st Division G.R. No. 4656 FACTS: Appeal by bill of exceptions. Spouses Miguel Ortiz and Calixta Felin died in Vigan, Ilocos Sur, in 1875 and 1882, respectively. Prior to her death, Calixta, executed, on August 17, 1876, a nuncupative will in Vigan, whereby she made her four children, named Manuel, Francisca, Vicenta, and Matilde, surnamed Ortiz y Felin, her sole and universal heirs of all her property. Manuel and Francisca were already deceased, leaving Vicenta and Matilda as heirs. In 1888, the defendants (Matilde and Gaspar), without judicial authorization, nor friendly or extrajudicial agreement, took upon themselves the administration and enjoyment of the properties left by Calixta and collected the rents, fruits, and products thereof, to the serious detriment of Vicentas interest. Despite repeated demands to divide the properties and the fruits accruing therefrom, Sps Gaspar and Matilde had been delaying the partition and delivery of the said properties by means of unkempt promises and other excuses. Vicenta filed a petition for partition with damages in the RTC. RTC decision: absolved Matilde from payment of damages. It held that the revenues and the expenses were compensated by the residence enjoyed by the

defendant party, that no losses or damages were either caused or suffered, nor likewise any other expense besides those aforementioned, Counsel for Matilde took an exception to the judgment and moved for a new trial on the grounds that the evidence presented did not warrant the judgment rendered and that the latter was contrary to law. That motion was denied by the lower court. Thus, this petition. ISSUE: WON a co-owner is required to pay for rent in exclusively using the coowned property. RULING: Article 394 of the Civil Code prescribes: Each co-owner may use the things owned in common, provided he uses them in accordance with their object and in such manner as not to injure the interests of the community nor prevent the co-owners from utilizing them according to their rights. Matilde Ortiz and her husband occupied the upper story, designed for use as a dwelling, in the house of joint ownership; but the record shows no proof that, by so doing, the said Matilde occasioned any detriment to the interests of the community property, nor that she prevented her sister Vicenta from utilizing the said upper story according to her rights. It is to be noted that the stores of the lower floor were rented and an accounting of the rents was duly made to the plaintiffs. Each co-owner of realty held pro indiviso exercises his rights over the whole property and may use and enjoy the same with no other limitation than that he shall not injure the interests of his coowners, for the reason that, until a division be made, the respective part of each holder can not be determined and every one of the coowners exercises together with his other coparticipants, joint ownership over the pro indiviso property, in addition to his use and enjoyment of the same. As the hereditary properties of the joint ownership of the two sisters, Vicenta Ortiz, plaintiff, and Matilde Ortiz, defendant, were situated in the Province of Ilocos Sur, and were in the care of the last named, assisted by her husband, while the plaintiff Vicenta with her husband was residing outside of the said province the greater part of the time between 1885 and 1905, when she left these Islands for Spain, it is not at all strange that delays and difficulties should have attended the efforts made to collect the rents and proceeds from the property held in common and to obtain a partition of the latter, especially during several years when, owing to the insurrection, the country was in a turmoil; and for this reason, aside from that founded on the right of co-ownership of the defendants, who took upon themselves the administration and care of the property of joint tenancy for purposes of their preservation and improvement, these latter are not obliged to pay to the plaintiff Vicenta one-half of the rents which might have been derived from the upper story of the said house on Calle Escolta, and, much less, because one of the living rooms and the storeroom thereof were used for the storage of some belongings and effects of common ownership between the litigants. The defendant Matilde, therefore, in occupying with her husband the upper floor of the said house, did not injure the interests of her coowner, her sister Vicenta, nor did she prevent the latter from living

therein, but merely exercised a legitimate right pertaining to her as a coowner of the property. MARIANO V. CA | GOSIENGFIAO, 222 SCRA 736- REDEMPTION BY A CO-OWNER Redemption of the whole property by a co-owner within the redemption period does not terminate the co-ownership and does not vest in him sole ownership. FACTS: Francisco Gosiengfaio is the registered owner of a parcel of land in Tuguegarao. In his lifetime, he mortgaged the land to Rural Bank of Tuguegarao to secure payment of a loan. Francisco died in without paying the debt. His intestate heirs were: his wife Antonia and children Amparo, Carlos, Severo, Grace, Emma, Ester, Francisco, Jr., Norma, Lina, and Jacinto. The bank foreclosed on the mortgage but before the redemption period expired, Antonia, Emma, Lina, Norma, Lina, Carlos and Severo executed a deed of assignment of the right of redemption in favor of Amparo. Amparo later on sold the land to Spouses Mariano. Grace Gosengfiao, and the other heirs excuded in the deed of assignment filed a complaint for recovery and legal redemption with damages against spouses Mariano. RTC decided in favor of spouses Mariano. CA for Grace Gosiengfia, et. al. ISSUE: Whether or not a co-owner who redeems the whole property with her own personal funds becomes the sole owner of said property and terminates the existing state of co-ownership? HELD: No. Admittedly, as the property in question was mortgaged by the decedent, a co-ownership existed among the heirs during the period given by law to redeem the foreclosed property. Redemption of the whole property by a co-owner does not vest in him sole ownership over said property but will inure to the benefit of all co-owners. In other words, it will not end to the existing state of coownership. Redemption is not a mode of terminating a co-ownership. Respondents have not lost their right to redeem, for in the absence of a written notification of the sale by the vendors, the 30-day period has not even begun to run. AGUILAR V. CA- CO-OWNERSHIP Any of the Co-owners may demand the sale of the house and lot at any time and the other cannot object to such demand; thereafter the proceeds of the sale shall be divided equally according to their respective interests. FACTS: Petitioner Vergilio and respondent Senen bought a house and lot in Paraaque where their father could spend and enjoy his remaining years in a peaceful neighborhood. They initially agreed that Vergilio will get 2/3 and Senen will get 1/3; but later they agreed on equal shares. Senen was left in the said lot to take care of their father since Vergilios family was in Cebu. After their fathers death petitioner demanded from private respondent that the latter vacate the house and that the property be sold and proceeds thereof divided among them but the latter refused. Petitioner then filed to compel the sale of the property. The

chunk of the issue tackled by the courts was regarding the pre-trial. Respondent filed a motion to cancel Pre-trial since the counsel had to accompany his wife in Dumaguete City where she would be a principal sponsor in a wedding. CFI denied the motion; and the pre-trial proceeded on the scheduled date. The respondents did not appear thus they were declared in default. The trial went on ex parte without the respondent and held that the property should be sold to a third party and that the proceeds be distributed to the parties; in addition respondent was made to pay rent from the time the action was filed. Respondents appealed this and the decision was reversed by the CA saying that the TC erred in declaring respondents in default; the case was then remanded to the trial court. Hence this appeal. ISSUE: A) W/N CA erred (1) in holding that the motion of respondent through counsel to cancel the pre-trial was dilatory in character and (2) in remanding the case to the trial court for pre-trial and trial? ISSUE RELEVANT TO PROPERTY: B) W/N trial court was correct with regards to the sale and rent? RULING: A) YES, CA erred in granting the respondents motion and remanding the case. The law is clear that the appearance of parties at the pretrial is mandatory. A party who fails to appear at a pre-trial conference may be non-suited or considered as in default. It is the discretion of the court to grant the motion if it sees that the reason for the cancelation of the same would be reasonable. SC found that the reason for the cancelation of the pre-trial was insufficient and that the trial court was not in grave abuse of discretion when they denied it. B) YES, with a few modification. Petitioner and respondents are co-owners of subject house and lot in equal shares; either one of them may demand the sale of the house and lot at any time and the other cannot object to such demand; thereafter the proceeds of the sale shall be divided equally according to their respective interests. BASIS: Article 494 of the Civil Code provides that no co-owner shall be obliged to remain in the co-ownership, and that each co-owner may demand at any time partition of the thing owned in common insofar as his share is concerned. Corollary to this rule, Art. 498 of the Code states that whenever the thing is essentially indivisible and the co-owners cannot agree that it be allotted to one of them who shall indemnify the others, it shall be sold and its proceeds accordingly distributed. SC held that of the proceeds should go to the petitioner and the remainder to the respondent (1,200 each.) Also rent was awarded 1,200 pesos per month with legal interest from the time the trial court ordered the respondent to vacate, for the use and enjoyment of the other half of the property. BASIS: When petitioner filed an action to compel the sale of the property and the trial court granted the petition and ordered the ejectment of respondent, the coownership was deemed terminated and the right to enjoy the possession jointly also ceased. OCAMPO et al vs. OCAMPO et al Leave a comment OCAMPO et al vs. OCAMPO et al G.R. No. 150707 April 14, 2004

FACTS: The complaint alleges that during the lifetime of the spouses Jose Ocampo and Juana Llander-Ocampo, they begot ten (10) children. 2 of them, Fidela, and Felicidad are respondents herein. The complaint further alleges that during the lifetime of the spouses Jose Ocampo and Luisa Llander-Ocampo, they acquired 3parcels of land and, upon their death, left the following properties. Only one of them, lot a is the subject of this case, a parcel of residential/ commercial land situated in the poblacion of Nabua, Camarines Sur that the 3 parcels of land are actually owned in common by the children of the late spouses although the land denominated as parcel (a) of the complaint is ostensibly registered in the name of Fidela Ocampo alone but acknowledged by her as a property owned in common by all of them, brothers and sisters; that plaintiffs desire to partition said properties but defendants Fidela Ocampo and Felicidad unlawfully and unreasonably refuse to do so; that the same defendants have been receiving the fruits of the properties to the exclusion of their co-heirs ;and, that because of their relationship, they undertook earnest efforts to amicably settle this controversy but because of defendants utterly unreasonable and unjustified actuations, the same failed. In their complaint, plaintiffs pray that judgment be rendered ordering the partition of the properties; ordering defendants Fidela and Felicidad to release or otherwise cancel any and all encumbrances which they had caused to be annotated on the TCT; requiring Fidela and Felicidad to refrain from further encumbering said properties; further ordering Fidela and Felicidad to indemnify plaintiffs . The Supplemental Complaint alleges that defendants Helen Ocampo-Barrito and Vicente Barrito are spouses; that in 1987, the TCT in the name of defendant Fidela and covering the lot described as parcel (a) was cancelled and, in lieu thereof aTCT was issued to defendant Belen Ocampo-Barrito, on the strength of an allege[d] Deed of Donation Inter Vivos ostensibly executed by defendant Fidela in their favor. That at the time the Deed of Donation Inter Vivos was presented for registration and when a TCT was issued to defendant Belen Ocampo-Barrito, both the donor and donees were notoriously aware that said properties were owned by the Ocampo brothers and sisters, and that the donor Fidela was not the exclusive owner thereof. The RTC holds and declares that defendant spouses are the true and lawful exclusive owners of the following properties. The CA affirmed with modifications (for damages) the said ruling. Hence this petition. ISSUE: At bottom, the question to be resolved in this case is who owns the disputed property? HELD: WHEREFORE, the Petition is hereby DENIED, and the assailed Decision AFFIRMED Petitioners chief evidence of co-ownership of the property in question is simply the Acknowledgement of Co-ownership executed by Fidela. As mentioned earlier, both the trial and the appellate courts were correct in finding that this

piece of documentary evidence could not prevail over the array of testimonial and documentary evidence that were adduced by respondents, such as: 1. On the other hand, Belen clearly traced the basis of her alleged sole ownership of the property and presented preponderant proof of her claim. she presented a Deed of Absolute Sale of Residential Land, referring to the subject property, executed between Adolfo Ocampo as seller and Felix Ocampo as buyer. The document dated in 1948, was acknowledged before a notary public. Likewise, in this Deed of Absolute Sale, Adolfo Ocampo declared his exclusive ownership of the property, having been acquired by purchase*;+ and *having+ been in [his] continuous, public, peaceful, adverse and material possession for more than 50 years together with [his] predecessors in rights and interest, in *the+ concept of owner without any claim of other persons.20 2. Respondent Belen proved that in 1953, this property had been sold to Fidela by Felix Ocampo for a valuable consideration; and that Fidela had entered the property, actually occupied it, and exercised all powers of dominion over it to the exclusion of petitioners. 3. To prove further that Fidela had exercised dominion over the property, Belen also presented a Real Estate Mortgage executed by the former as absolute owner. Fidela had executed it in favor of her sister Apolonia Ocampo, one of the original petitioners in this case, who is now represented by her heirs. Belen correctly argues that in agreeing to be a mortgagee, Apolonia admitted and recognized Fidela as the true owner of the land in question. 4. Belen then presented a Deed of Donation Inter Vivos executed in 1984, between herself as donee and Fidela as donor. This act shows the immediate source of the formers claim of sole ownership of the property 5. In addition to the TCT presented, Belen offered as evidence the Tax Declaration indicating that she, as owner, had been paying real estate taxes on the property, all to the exclusion of petitioners. The Civil Code provides that an essential requisite of a contract of mortgage is that the mortgagor be the absolute owner of the thing mortgaged. Co-ownership cannot be presumed even if only a portion of the property was mortgaged to Apolonia, because a co-owner may dispose only of ones interest in the ideal or abstract part of the undivided thing co-owned with others. The effect of a mortgage by a co-owner shall be limited to the portion that may be allotted to that person upon the termination of the co-ownership. In this case, Fidela mortgaged a definite portion of the property and thus negated any acknowledgement of co-ownership. A donation as a mode of acquiring ownership results in an effective transfer of title to the property from the donor to the donee. Petitioners stubbornly rely on the Acknowledgement of Co-ownership allegedly executed by Fidela in favor of her siblings. What they overlook is the fact that at the time of the execution of the Acknowledgement assuming that its authenticity and due execution were proven the property had already been donated to Belen. The Deed of Donation, which is the prior document, is clearly inconsistent with the document (Acknowledgement of Co-ownership) relied upon by petitioners. On the other hand, petitioners could not show any title, tax receipt or document to prove their ownership. Having filed an action involving property, they should

have relied on the strength of their own title and not on the alleged weakness of respondents claim. Neither can we accept petitioners contention that co-ownership is shown by the fact that some of the children of Spouses Ocampo stayed, lived, and even put up businesses on the property. The appellate court correctly found that since the litigants in this case were blood relatives, fraternal affection could have been a good motive that impelled either Belen or Fidela to allow petitioners to use the property. Without any proof, however, co-ownership among the parties cannot be presumed. It is quite surprising that despite the process of transfers and titling of the subject property commencing in 1948 and eventually leading to the sole ownership of Belen in 1984 it was only after 1984 that petitioners started asserting their claim of co-ownership thereof NOTES: 1. Petitioners argue that the Acknowledgement of Co-ownership may be considered as a declaration against interest. A statement may be admissible as such a declaration if it complies with the following requisites: 1) the declarant is dead or unable to testify; 2) it relates to a fact against the interest of the declarant; 3) at the time of the declaration, the declarant was aware that it was contrary to his or her interest; and 4) the declarant had no motive to falsify and believed the declaration to be true The Acknowledgement of Co-ownership could not be a fact against the interest of the declarant Fidela, since her right over the property had already been extinguished by the prior act of donation. Thus, at the time of the declaration, Fidela could not have acknowledged co-ownership, as she had no more property against which she had an interest to declare. 2. Donation is an act of liberality whereby a person gratuitously disposes of a thing or a right in favor of another who accepts it. Once perfected, a donation is final; its revocation or rescission cannot be effected, absent any legal ground therefor. A donation may in fact comprehend the entire property of the donor. At any rate, the law provides that donors should reserve, in full ownership or in usufruct, sufficient means for their own support and that of all their relatives who, at the time of the acceptance of the donation, are by law entitled to be supported by them. 3. To be sure, petitioners arguments all pertain to circumstances extraneous to the Deed of Donation itself. The law is clear that when its terms have been reduced to writing, an agreement must be presumed to contain all the terms agreed upon; and there can be, between the parties and their successors in interest, no evidence of such terms other than the contents of the written agreement] SULPICIO CARVAJAL, petitioner, vs. THE HONORABLE COURT OF APPEALS ** and EUTIQUIANO CAMARILLO and LIBERATA CACABELOS, respondents. Jul

25 Facts: EUTIQUIANO CAMARILLO and LIBERATA CACABELOS who are husband and wife, had instituted a complaint before the CFI for ejectment and recovery of possession against SULPICIO CARVAJAL, alleging that they are the owners of a parcel of commercial land, pro-indiviso, consisting of 150.8 sq. meters, more or less, situated in Poblacion, Tayug, Pangasinan, having bought the same from Evaristo G. Espique by virtue of a Deed of Absolute Sale executed on April 15, 1964. They also demand that petitioner pay a monthly rental for the use of the property all P40.00 until the property is surrendered to them. The property in question is a 1/5 portion of a 754 sq. meter land originally owned by Hermogenes Espique and his wife, both dead. After their death their five children, namely: Maria, Evaristo, Faustina, Estefanio and Tropinia succeeded them in the ownership of the whole lot. Petitioner occupies two-fifths of the whole lot inherited pro-indiviso by the Espique children. Petitioner alleges that he purchased the northern one-half portion of the lot he is occupying (which is also claimed by respondents) from Estefanio Espique and that the southern one-half portion of the lot he is occupying (which is also claimed by respondents) from Estefanio Espique and that the southern one-half portion is leased to him by Tropinia Espique. The land subject of the controversy is the most southern portion of the whole lot inherited by the Espique children which petitioner claims he had bought from Estefanio on April 26, 1967 and which respondents claim they had bought from Evaristo on April 15, 1964. Both sales were made while the petition for partition filed by Evaristo Espique was still pending before the Court of First Instance of Pangasinan. Held: The respondents have no right to eject petitioners nor demand payment of rentals for the use of the property in dispute. Until the partition of the estate is ordered by the Court of First Instance of Pangasinan in the pending partition proceedings and the share of each co-heir is determined by metes and bounds, neither petitioner nor respondents can rightfully claim that what they bought is the part in dispute. Ratio: Under Article 493 of the New Civil Code, each co-owner shall have the full ownership of his part and of the fruits and benefits pertaining thereto and he may alienate, assign or mortgage it, and even substitute another person in its enjoyment, the effect of the alienation or the mortgage with respect to the coowners, shall be limited, by mandate of the same article, to the portion which may be allotted to him in the division upon the termination of the co-ownership. He has no right to sell or alienate a concrete, specific, or determinate part of the thing in common to the exclusion of the other co-owners because his right over the thing is represented by an abstract or ideal portion without any physical adjudication. An individual co- owner cannot adjudicate to himself or claim title to any definite portion of the land or thing owned in common until its actual partition by agreement or judicial decree. Prior to that time all that the co-owner has is an Ideal or abstract quota or proportionate share in the entire thing owned in common by all the co-owners. What a co owner may dispose of is only his undivided aliquot share, which shall be limited to the portion that may be allotted to him upon partition. Before partition, a co-heir can only sell his successional rights.

In the case at bar, the fact that the sale executed by Evaristo G. Espique in favor of respondents and the sale executed by Estefanio Espique in favor of petitioner were made before the partition of the property among the co-heirs does not annul or invalidate the deeds of sale and both sales are valid. However, the interests thereby acquired by petitioner and respondents are limited only to the parts that may be ultimately assigned to Estefanio and Evaristo, respectively, upon the partition of the estate subject to provisions on subrogation of the other co-heirs to the rights of the stranger-purchaser provided in Article 1088 of the Civil Code. Respondent courts ruling that the sale by Estefanio in favor of petitioner is not valid because of lack of notice to his co-heirs is erroneous. Such notice in writing is not a requisite for the validity of the sale. Its purpose is merely to apprise the co-heirs of the sale of a portion of the estate, for them to exercise their preferential right of subrogation under Article 1088 of the New Civil Code, that is, the right to redeem the property sold within one month from the time they were notified in writing of the sale by a co-heir. REPUBLIC VS. HEIRS OF FRANCISCA DIGNOS-SORONO Leave a comment REPUBLIC VS. HEIRS OF FRANCISCA DIGNOS-SORONO G.R. No. 171571 March 24, 2008 FACTS: 2 were adjudicated by the then Court of First Instance of Cebu in favor of the following in four equal shares: a) Francisca Dignos, married to Blas Sorono b) Tito Dignos share in the two lots; c) predecessors-in-interest of the respondents and d) predecessors-in-interest of the respondents share in the two lots share in the two lots; share in the two lots;

Lapu-lapu City. Respondents further alleged that neither they nor their predecessors-in-interests sold, alienated or disposed of their shares in the lots of which they have been in continuous peaceful possession. Respondents furthermore alleged that neither petitioner nor its predecessor-in-interest had given them any written notice of its acquisition of the share of Tito Dignos.

The Republic, represented by the MCIAA in its Answer with Counterclaim, maintained that from the time the lots were sold to its predecessor-in-interest CAA, it has been in open, continuous, exclusive, and notorious possession thereof; through acquisitive prescription, it had acquired valid title to the lots since it was a purchaser in good faith and for value; and assuming arguendo that it did not have just title, it had, by possession for over 30 years, acquired ownership thereof by extraordinary prescription. At all events, petitioner contended that respondents action was barred by estoppel and laches.

The trial court found for respondents. the CA affirmed the trial courts decision. Hence, the present petition for review on certiorari

ISSUE:

1. WON the sale of the entire 2 lots by the heirs of Tito binding to the respondents

2. WON estoppel and laches should work against respondents

HELD: the petition is denied

It appears that the two lots were not partitioned by the adjudicatees. 1. NO. Article 493 of the Civil Code provides: It appears further that the heirs of Tito Dignos, who was awarded share in the two lots, sold the entire two lots to the then Civil Aeronautics Administration (CAA) via a public instrument entitled Extrajudicial Settlement and Sale without the knowledge of respondents whose predecessors-in-interest were the adjudicatees of the rest of the portion of the two lots. In 1996, CAAs successor-in-interest, the Mactan Cebu International Airport Authority (MCIAA), erected a security fence one of the lot and relocated a number of families, who had built their dwellings within the airport perimeter, to a portion of said lot to enhance airport security. MCIAA later caused the issuance in its name of a Tax Declarations of the 2 lots. Respondents soon asked the agents of MCIAA to cease giving third persons permission to occupy the lots but the same was ignored. Respondents thereupon filed a Complaint for Quieting of Title, Legal Redemption with Prayer for a Writ of Preliminary Injunction against MCIAA before the RTC of

Each co-owner shall have the full ownership of his part and of the fruits and benefits pertaining thereto, and he may therefore alienate, assign or mortgage it, and even substitute another person in its enjoyment, except when personal rights are involved. But the effect of the alienation of the mortgage, with respect to the co-owners, shall be limited to the portion which may be allotted to him in the division upon the termination of the co-ownership.

Apropos is the following pertinent portion of this Courts decision in BailonCasilao v. CA:

As early as 1923, this Court has ruled that even if a co-owner sells the whole property as his, the sale will affect only his own share but not those of the other co-owners who did not consent to thesale.This is because under the aforementioned codal provision, the sale or other disposition affects only his undivided share and the transferee gets only what would correspond to his grantor in the partition of the thing owned in common.

the sale by the vendor. The Court may take judicial notice of the increase in value of the lots.

As mentioned earlier, however, the heirs of Tito Dignos did not notify respondents about the sale. At any rate, since the Extrajudicial Settlement and Sale stipulates, thus:

rebutted and proven otherwise. Joaquino, having failed to prove that she was financially capable and that she purchased said properties in her exclusive capacity, could not make a valid claim of ownership.As to Joaquino's claim of having the benefit of co-ownership conferred by the common-law relationship under Article 144 of the Civil Code (in connection with Article 148 of the Family Code), the Court reiterated that the said provision is inapplicable to common-law relations amounting to adultery or concubinage. Jurisprudence holds that for Article 144 to apply, the couple must not have any legalimpediment to contract a marriage. And since Rodolfo and Joaquino were incapacitated to marry due to Rodolfo's marriage with Lourdes, she cannot validly invokethe right conferred. PLEASANTVILLE DEVELOPMENT CORPORATION VS. COURT OF APPEALSG.R. NO. 79688 253 SCRA 10 FEBRUARY 1, 1996PONENTE: PANGANIBAN, J. Doctrine: Good faith consists in the belief of the builder that he land he is building on is his and hisignorance of any defect or flaw in his title. The burden of proving bad faith belongs to the one asserting it. Facts: Edith Robillo purchased from Pleasantville Development Corporation, herein petitioner a parcel ofland at Pleasantville Subdivision, Bacolod City. The property was designated as Lot 9, Phase II. In 1975,herein respondent Eldred Jardinico bought the said subject lot from the former purchaser. Eldred laterdiscovered that the property he purchased had improvements introduced therein by respondent WilsonKee.Kee on the other hand bought on installments Lot 8 of the same subdivision from C.T. TorresEnterprises, Inc. (CTTEI) which is the exclusive real estate agent of the petitioner. Under the contract Keewas allowed to take possession of the property even before full payment of the price. CTTEI through an employee, Zenaida Octaviano accompanied Kees wife Donabelle to inspec t Lot No. 8. Octavianohowever mistakenly pointed towards Lot 9. Hence spouses Kee had their residence, an auto repair shop,a store and other improvements constructed on the wrong lot.Upon discovery of the blunder both Kee and Jardinico tried to reach an amicable settlement butthey failed. Jardinico demanded that the improvements be removed but as Kee refused, Jardinico filed acomplaint for ejectment with damages against Kee at the Municipal Trial Court in Cities (MTCC) ofBacolod City. Kee filed a third-party complaint against herein petitioner and CTTEI.The MTCC found that the error was attributable to CTTEI also since at present the contract with Kee has rescinded for Kees failure to pay installments. Kee no longer had any right over the subje ctproperty and must pay rentals for its use. The Regional Trial Court (RTC) of Bacolod City ruled thatpetitioner and CTTEI were not at fault or were not negligent. It argued that Kee was a builder in bad faith.Even if assuming that he was in good faith, he was no longer so and must pay rentals from the time thathe was given notice to vacate the lot. The Court of Appeals ruled that Kee was a builder in good faith ashe was unaware of the mix-up when he constructed the improvements. It was in fact due to thenegligence and wrongful delivery of CTTEI which included its principal the herein petitioner. It furtherruled that the award of rental was without basis.Pending the resolution of the case at the Court of Appeals Jardinico and Kee entered into a deedof sale, wherein Lot 9 was sold to Kee. In the said deed a provision stating that regardless of the outcomeof the decision, such shall not be pursued by the parties and shall be considered dismissed and withouteffect. The appellate court was not informed of this deal. Issue:

From the foregoing, it may be deduced that since a co-owner is entitled to sell his undivided share, a sale of the entire property by one co-owner without the consent of the other co-owners is NOT null and void. However, only the rights of the co-owner-seller are transferred, thereby making the buyer a co-owner of the property.

That the HEIRS-VENDORS, their heirs, assigns and successors, undertake and agree to warrant and defend the possession and ownership of the property/ies herein sold against any and all just claims of all persons whomsoever and should the VENDEE be disturbed in its possession, to prosecute and defend the same in the Courts of Justice.

Petitioners predecessor-in-interest CAA thus acquired only the rights pertaining to the sellers-heirs of Tito Dignos, which is only undivided share of the two lots.

2. NO. Registered lands cannot be the subject of acquisitive prescription. Petitioners insistence that it acquired the property through acquisitive prescription, if not ordinary, then extraordinary, does not lie. It bears emphasis at this juncture that in the Extrajudicial Settlement and Sale forged by CAA and Tito Dignos heirs the following material portions thereof validate the claim of respondents that the two lots were registered: x x x x That since the OCT of Title of the above-mentioned property/ies has/have been lost and/or destroyed and the VENDEE hereby binds itself to reconstitute said title/s at its own expense and that the HEIRS-VENDORS, their heirs, successors and assigns bind themselves to help in the reconstitution of title so that the said lot/s may be registered in the name of the VENDEE in accordance with law x x x x NOTES: As for petitioners argument that the redemption price should be of the prevailing market value, not of the actual purchase price, since, so it claims, (1) the respondents received just compensation for the property at the time it was purchased by the Government; and, (2) the property, due to improvements introduced by petitioner in its vicinity, is now worth several hundreds of millions of pesos, the law is not on its side.

Thus, Article 1088 of the Civil Code provides:

Should any of the heirs sell his hereditary rights to a stranger before the partition, any or all of the co-heirs may be subrogated to the rights of the purchaser by reimbursing him for the price of the sale, provided they do so within the period of one month from the time they were notified in writing of

Petitioner is not without any remedy. This decision is, therefore, without prejudice to petitioners right to seek redress against the vendors-heirs of Tito Dignos and their successors-in-interest. Joaquino vs. ReyesFacts:Lourdes Reyes was the widow of Rodolfo Reyes, having been married in 1947 in Manila. Rodolfo, however, in the course of their marriage, had illicit relations with one Milagros Joaquino, to whom he allegedly "put into custody" some of the couple's conjugal properties. Said properties specifically include his earnings and retirement benefits from working as the Vice Presidet and Comptroller of Warner Barns and two cars; and that the amount herein stated was used to pay off theloan and monthly mortgage of a house in Paranaque, registered under Joaquino's name. Lourdes then prayed that the properties be declared conjugal, that Milagrossurrenders the possession thereof, and that damages be awarded. Milagros, on the other hand, contends that she purchased the mentioned properties in her exclusive capacity, that she had no knowledge of the Rodolfo's first marriage, that she had was never a beneficiary of the latter's earnings, and that her living together with Rodolfo for nineteen (19) years, along with the fact that she had children with him, be considered by the court in rendering judgment.Lourdes, however, died and was later represented by her children with Rodolfo. Subsequently, the trial court granted Lourdes' complaint. Upon appeal to the CA,however, Milagros reiterated her stand and questioned the findings of the trialcourt. But to no avail, the CA likewise held that the property had been paid outof the conjugal funds of Rodolfo and Lourdes, because the funds used to pay thehouse off was sourced from Rodolfo's earnings as part of the conjugal partnership.Issue:WON the properties in question were conjugalWON the petitioner's common-law relationship with Rodolfo validates her claim ofownershipHeld:Yes. The property regime applicable is the CPG, having been the default propertyregime during the time of Lourdes' marriage. Such properties include the following, as enumerated by Article 153:(1) That which is acquired by onerous title during the marriage at the expense of the common fund, whether the acquisition be for the partnership, or for only one of the spouses;(2) That which is obtained by the industry, or work, or as salary of the spouses, or of either of them;(3) The fruits, rents or interests received or due during the marriage, coming from the common property or from the exclusive property of each spouse. Subject properties fall squarely within the said categories.Article 160 then prescribes that all properties of the marriage are presumed tobe conjugal and covered by the CPG unless

Whether or not a lot buyer who constructs improvements on the wrong property erroneously delivered by the owners agent, a builder in good faith? Held: Yes. Article 527 of the Civil Code provides the presumption that petitioner has the burden ofproving that Kee was a builder in bad faith. Kee may be made liable for the violation of the contract withCTTEI but this may not be used as a basis of bad faith and as a sufficient ground to negate thepresumption of good faith. Jardinico is presently only allowed to file a complaint for unlawful detainer.Good faith is based on the belief of the builder that the land he is building on is his and his ignorance ofany flaw or defect in is title. Since at the time when Kee constructed his improvements on Lot 8, he wasnot aware that it was actually Lot 9 that was delivered to him. Petitioner further contends that Kee wasnegligent as a provision in the Contract of Sale on Installment stated that the vendee must have Banco Espanol Filipino v Peterson (1907)FactsOn March 4, 1905, Banco Espanol Filipino (BEP) executed a contract of loan infavor of Francisco Reyes for P141 702.00. Reyes was already indebted to thebank for P84 415.00. His total debt was therefore P226 117.38. To secure payment of the P141k and the P84k, Reyes executed a publicinstrument1. Mortgaging several of his properties2. Pledging part of his personal property to BEP (P90 591.75 worth of wines,liquors and canned goods), which were stored at a warehouse he rented inManilaBEP and Reyes agreed that the goods should be delivered to Ramon Garcia(depositary) for safekeeping. Reyes turned over the goods to R. Garcia bygiving him the warehouse keys.On September 29, 1905, BEP and Reyes substituted Luis Sierra in place of R.Garcia as the depositary.On October 19, 1905, Juan Garcia (yes, related to Ramon) brought an actionagainst Francisco Reyes and Ramon Agtarat. CFI Manila ruled against Reyesand Agtarat for P15 000.00.On the same day, Sheriff James Peterson entered the warehouse where thegoods pledged to BEP were stored under the custody of the depositary, Sierra.Peterson levied upon P30 000 worth of the goods pledged to the bank,depriving the latter of possession of the same, as stipulated in the March 4contract of loan.IssuesWas the contract of pledge between BEP and Reyes to secure a loan valid?Was Reyes still in possession of the pledged property, thereby making thecontract defective?Held The contract was valid. Reyes was no longer in possession of the pledgedproperty. BEP had symbolic possession of the same. The contract complies with all the requisites of a valid pledge contract, asprescribed by the Civil Code: 1. The property was pledged to secure a debt2. The date of execution, the terms of the pledge, and the property pledgedappeared in a public instrument3. The property pledged was placed in the hands of a third person (in thiscase, Sierra) by common consent of the debtor and creditor, under thesupervision of an agent (in this case, Rodriguez) of the bankReyes, after the pledge, parted with the possession of his personal property,which was delivered to a third person (R. Garcia, and subsequently, Sierra)who would take care of them for BEP.Sierra was the third person appointed by common consent of BEP (creditor)and Reyes (debtor), to hold possession over the goods pledged in favor of thebank under the direct supervision of Rodriguez, an agent specificallyappointed by the bank. The contract in question was, therefore, a perfect contract of pledge underarticles 1857 and 1863 of the Civil Code, it having been conclusively shownthat the pledgee (BEP) took charge and possession of the goods pledgedthrough a depositary (Sierra) and a special agent (Rodriguez) appointed by it,each of whom had a duplicate key to the warehouse wherein the said goodswere stored, and that the pledgee (BEP), itself, received and collected theproceeds of the goods as they were sold. The legality of the

pledge was not affected by the fact that the goodsremained in the warehouse formerly rented by Reyes the pledgor. This isbecause after the pledge had been agreed upon, and after the depositoryappointed with common consent of the parties had taken possession of thesaid property, Reyes could no longer dispose of the same because BEP wasthe only party allowed to do so through Sierra and Rodriguez. The symbolic transfer of the goods through delivery of the keys to thewarehouse where the goods were stored was sufficient evidence to show thatSierra, the depositary appointed by both BEP and Rodriguez, was legallyplaced in possession of the goods.Since the contract of pledge was valid, BEP had a better right to the goodscompared to J. Garcia. The Court ordered either the return of the improperlylevied goods, or the payment of their value, P30 000 Easement Right of Way Public Highway Benedicto owns Hacienda Toreno which is located in Victorias, Negros Occidental. Two roads pass through the said hacienda: the Dacuman-Toreno Road and the Nanca-Victorias Road. For forty years, the owners of the nearby hacienda, Cuaycong et al, had been using the said roads to transport their products. But in 1911, Benedicto decided to close the road and began asking for toll fees for wagons passing through their hacienda. In 1912, Cuaycong et al sued Benedicto. Cuaycong claimed that they have a right of way over the said NancaVictorias Road considering that they have been using it since time immemorial. The lower court dismissed the claim over the Dacuman-Toreno Road for the other parties were in default, but the lower court declared that Cuaycong et al do have a right of way over the Nanca-Victorias Road. Benedicto appealed. Cuaycong then averred that the road is a public highway. ISSUE: Whether or not Cuaycong et al were able to establish their right over the Nanca-Victorias Road. HELD: No. The Nanca-Victorias Road is not a public highway. First it was shown that in the Torrens title held by Benedicto, there was no encumbrance attached to the hacienda, that it is nowhere nearby a road nor does it border a road. Second, the road was not maintained by the local government. Its upkeep was solely supported by the road users for their benefits, convenience and interest. There was no adverse possession by the government. Third, there was no evidence which shows that the land is of the nature of a public highway. It was shown that the road was in existence since 1885, but it was not shown as a public highway, in fact, the other evidence shown pertain to Dacuman-Toreno Road. Fourth, the road was closed in 1911; it was only in 1912 that Cuaycong et al filed their suit. Neither did Cuaycong et al acquired a right of private easement. The lower court ruled that Cuaycong et al and their predecessors in interest had been using the said road since time immemorial yet they only showed evidence that it was in use in 1885 but no other evidence to show a further time of usage was ever shown to prove their claim. Bogo-Medellin Milling Co., Inc. v CA G.R. No. 124699 July 31, 2003 FACTS: The respondents in this case were the heirs of Magdaleno Valdez Sr., who purchased an unregistered parcel of land located in Cebu from FelicianaSantillan (seller). The land was possessed by decedent who had also paidtaxes thereon. The heirs subsequently inherited the land. However, asugar company, BogoMedellin Milling Co. was able to obtain title to Lot No. 954, the narrow lot where

the railroad tracks (existent even prior tothe sale to decedent) lay. The lot was likewise declared for tax purposesunder the name of the company.The heirs filed a complaint for Compensation and/or Recovery of Possession of the lot claiming that Bomedco was granted by the seller of the lot a railroad right of way for a period of 30 years which had expiredsometime in 1959 but that the heirs allowed Bomedco to continue usingthe land because one of them was then an employee of the company.Bomedco, on the other hand, claimed that it was the owner and possessor of the registered lot when it bought the lot from seller in 1929 and that theheirs were already barred by prescription and laches because of Bomedcos open and continuous possession of the property for more than 50 years.The trial court rejected the evidence presented by Bomedco (as it wasonly a Xerox copy of an unsigned deed of Sale) but ruled that Bomedcohad already acquired ownership of the property through acquisitive prescription because it possessed the property in good faith for more than10 years. This was reversed by the Court of Appeals which ruled thatBomedco only acquired an easement of right of way by unopposed andcontinuous use of the land, but not ownership. .ISSUE: 1) whether Bomedco had indeed acquired ownership of the land throughextraordinary acquisitive prescription? 2) Whether easement was continuous and thus Bomedco hadacquired title over the use of the land? RATIO/HELD: 1. No. Bomedco only had a right of easement over the land as shown bytax receipts wherein it declared, for several years, the property to be a central railroad right of way or sugar railroad right of way when itcould have declared it to be industrial land as it did for the years 1975 and 1985. Instead of indicating ownership of the lot, these receiptsshowed that all petitioner had was possession by virtue of the right of waygranted to it. X x x A person cannot have an easement on his own land,since all of the uses of an easement are fully comprehended in his generalright of ownership. An easement or servitude is a real right, constituted on the corporealimmovable property of another, by virtue of which the owner has torefrain from doing, or must allow someone to do something on hisproperty, for the benefit of another thing or person. It exists onlywhen the servient and dominant estates belong to two differentowners. It gives the holder of the easement an incorporeal interest onthe land but grants no title thereto. Therefore, an acknowledgment of the easement is an admission that the property belongs to another. Having held the property by virtue of an easement, Bomedco cannot nowassert that its occupancy since 1929 was in the concept of an owner. Neither can it declare that the 30- year period of extraordinary prescription started from that year.Moreover, the mere expiration of the period of easement in 1959 did not convert petitioners possession into an adverse one. Mere material possession of land is not adverse possession as against the owner and isinsufficient to vest title, unless such possession is accompanied by theintent to possess as an owner.2. An easement is continuous if its use is, or may be, incessant withoutthe intervention of any act of man, like the easement of draineage andit is discontinuous if it is used at intervals and depends on the act of man, like the easement of right of way. x x x an easement of right of way of railroad tracks is discontinuous because the right is exercised only if and when a train operated by a person passes ove r anothers property.

A party is deemed to acquire title over the use of the land if: a) it hadsubsequently entered into a contractual right of way with the heirs for thecontinued use of the land under the principles of voluntary easements,or b) it had filed a case against the heirs for conferment on it of a legaleasement of right of way (see orig case for the requirements)The point is, bomedco did not exercise any of the abovementionedoptions in order for it to acquire title over the railroad right of way GARGANTOS V. CA- EASEMENT FACTS: Sanz was the previous owner of a land which he subdivided into several lots. One lot was sold to Tengtio, whol sold to Uy Veza. Another lot with a house constituted thereon was sold to Tan Yanon. A third portion with a warehouse was sold to Gargantos. The problem arose when latter asked from the Municipality for a permit to demolish the warehouse in order to construct a higher one. Yan Yung opposed for it would block his window and impair his right of loght and view. ISSUE: Whether or not an easement was established RULING: Yes. Again, Art. 624 provides that when two adjoining estates were formerly owned by one person who introduced improvements on both such that the wall of the house contructed on the first estate extends to the wall of the warehouse on the second estate; and at the time of the sale of the first estate, there existed on the aforementioned wall of the house, doors, windows which serve as passages for light and view, there being no provision in the deed of sale that the easement of light and view will not be established, the apparent sign of easement between the two estates is established as a title. VALDES Facts: Spouses Victor and Jocelyn Valdez purchased from spouses Francisco Tabisula and Caridad Tabisula a parcel of land. Contained in the deed of sale is a stipulation that the Sps. Valdez shall be provided a 2 1/2 meters *sic+ wide road right-of-way on the western side of their lot but which is not included in this sale. Sps. Tabisula then built a concrete wall on the subject property. Feeling betrayed by said act of Sps. Tabisula based on the deed of sales intended road right of way, Sps. Valdez reported the matter the brgy. Lupon but it was in vain which constrained Sps. Valdez to file a case for specific performance against the Tabisulas with the RTC. The Sps. Tabisula contended that :1: Sps. Valdez and family also are the owners of two properties adjoining the subject property, which adjoining properties have access to two public roads ; and 2: they could not have agreed to providing petitioners an easement on the western side of their lot as there exists a two-storey concrete house on their lot where the supposed easement is to be located, which was erected long before the subject property was sold to the Valdezs; thus , the easement should be taken from the western portion of the subject property and not from theirs. RTC dismissed the Sps. Valdezs complain. On appeal, Sps. Valdez were again turned down. Issue: WON the Sps. Valdez are entitled to the right of way as provided for in the deed of sale Ruling: The Sps. Valdez are not entitled to the right of way.

An easement or servitude is a real right constituted on anothers property, corporeal and immovable, by virtue of which the owner of the same has to abstain from doing or to allow somebody else to do something on his property for the benefit of another thing or person. The statutory basis of this right is Article 613 of the Civil Code which reads: Art. 613. An easement or servitude is an encumbrance imposed upon an immovable for the benefit of another immovable belonging to a different owner. The immovable in favor of which the easement is established is called the dominant estate; that which is subject thereto, the servient estate. There are two kinds of easements according to source by law or by the will of the owners. So Article 619 of the Civil Code provides: Art. 619. Easements are established either by law or by the will of the owners. The former are called legal and the latter voluntary easements. From the allegations in Sps Valdez complaint, it is clear that what they seek to enforce is an alleged grant in the deed by respondents of an easement reading: they shall be provided a 2 meters wide road right-of-way on the western side of their lot but which is not included in this sale. Article 1358 of the Civil Code provides that any transaction involving the sale or disposition of real property must be in writing. The stipulation harped upon by petitioners that they shall be provided a 2 meters wide road right-of-way on the western side of their lot but which is not included in this sale is not a disposition of real property. The proviso that the intended grant of right of way is not included in this sale could only mean that the parties would have to enter into a separate and distinct agreement for the purpose. The use of the word shall, which is imperative or mandatory in its ordinary signification, should be construed as merely permissive where, as in the case at bar, no public benefit or private right requires it to be given an imperative meaning. Besides, a document stipulating a voluntary easement must be recorded in the Registry of Property in order not to prejudice third parties. So Articles 708 and 709 of the Civil Code call for, viz: Art. 708. The Registry of Property has for its object the inscription or annotation of acts and contracts relating to the ownership and other rights over immovable property. Art. 709. The titles of ownership, or of other rights over immovable property, which are not duly inscribed or annotated in the Registry of Property shall not prejudice third persons. Sps Valdez are neither entitled to a legal or compulsory easement of right of way. For to be entitled to such kind of easement, the preconditions under Articles 649 and 650 of the Civil Code must be established, viz: Art. 649. The owner, or any person who by virtue of a real right may cultivate or use any immovable, which is surrounded by other immovables pertaining to other persons, and without adequate outlet to a public highway, is entitled to demand a right of way through the neighboring estates, after payment of the proper indemnity. xxxx This easement is not compulsory if the isolation of the immovable is due to the proprietors own acts. (Underscoring supplied) Art. 650. The easement of right of way shall be established at the point least prejudicial to the servient estate, and, insofar as consistent with this rule, where the distance from the dominant estate to a public highway may be the shortest. (Underscoring supplied) Thus, to be conferred a legal easement of right of way under Article 649, the following requisites must be complied with: (1) the property is surrounded by other immovables and has no adequate outlet to a public highway; (2) proper indemnity must be paid; (3) the isolation is not the result of the owner of the

dominant estates own acts; (4) the right of way claimed is at the point least prejudicial to the servient estate; and (5) to the extent consistent with the foregoing rule, the distance from the dominant estate to a public highway may be the shortest. The onus of proving the existence of these prerequisites lies on the owner of the dominant estate, herein Sps. Valdez. Since the Sps. Valdez then have more than adequate passage to two public roads, they have no right to demand the grant by the Sps. Tabisula of an easement on the western side of the Tabisulas lot.; it appearing that the Sps. Valdez and their family are also the owners of two properties adjoining the subject property which have access to two public roads or highways. Napocor vs Purefoods Corp., Solid Development Corporation, Jose Ortega Jr., Silvestre Bautista, Alfredo Cabande, Heirs of Trinidad, Moldex Realty Inc. G.R. No. 160725, September 12 2008 [Eminent domain; determination of just compensation] FACTS: Napocor sought to acquire an easement of right-of-way over certain parcels of land situated in four towns of the province of Bulacan for the construction and maintenance of its 500 KV Transmission Line project in Northern Luzon. It filed a special civic action for eminent domain before the trial court against the registered owners or claimants of parcel of lands affected. The complaint alleged the public purpose of the Northwestern Luzon Project, as well as the urgency and necessity of acquiring easements of right-of-way over the said parcels of land. Only PFI, SDC, Moldex and the heirs of Trinidad filed their respective answers raising the issue of just compensation of their property to be expropriated. A report submitted to the RTC recommending that the compensation due from NAPOCOR be based on the fair market value of P 600/sq m for properties belonging to Moldex and P 400/sq m for properties belonging to the rest of the respondents. RTC rendered a Decision based on the report, ordering payment of just compensation by Napocor to name respondents with legal interest of 6%/annum until finality of the Decision and at 12%/annum from its finality until full payment. CA affirmed RTC decision in all respects except for the period during which the interest of 12% per annum would accrue. NAPOCOR assailed the CA's reliance on the commissioners report in fixing just compensation based on the full market value of the affected properties. NAPOCOR contends that only an easement of right-of-way for the construction of the transmission line project is being claimed, thus, only an easement fee equivalent to 10% of the fair market value of the properties should be paid to the affected property owners (Section 3A, R.A. 6395, as amended and the implementing regulation of R.A. No. 8974). ISSUE: Whether or not only an easement fee of 10% of the market value of the expropriated properties should be paid to the affected owners. RULING: No. The Court ruled that the just compensation in the amount of only 10% of the market value of the property was not enough to indemnify the incursion on the affected property.

Expropriation is not limited to the acquisition of real property with a corresponding transfer of title or possession. The right-of-way easement resulting in a restriction or limitation of property rights over the land traversed by transmission lines also falls within the ambit of the term "expropriation." In eminent domain or expropriation proceedings, the general rule is that the just compensation to which the owner of the condemned property is entitled is the market value. The aforementioned rule, however, is modified where only a part of a certain property is expropriated. In addition to the market value of the portion taken, he is also entitled to recover the consequential damage, if any, to the remaining part of the property. At the same time, from the total compensation must be deducted the value of the consequential benefits.

and from Lucena City are thus compelled to close down their existing terminals and use the facilities of Lucena. The true role of Constitutional Law is to effect an equilibrium between authority and liberty so that rights are exercised within the framework of the law and the laws are enacted with due deference to rights. A due deference to the rights of the individual thus requires a more careful formulation of solutions to societal problems. From the memorandum filed before the Court by Lucena, it is gathered that the Sangguniang Panlungsod had identified the cause of traffic congestion to be the indiscriminate loading and unloading of passengers by buses on the streets of the city proper, hence, the conclusion that the terminals contributed to the proliferation of buses obstructing traffic on the city streets. Bus terminals per se do not, however, impede or help impede the flow of traffic. How the outright proscription against the existence of all terminals, apart from that franchised to Lucena, can be considered as reasonably necessary to solve the traffic problem, the Court has not been enlightened. If terminals lack adequate space such that bus drivers are compelled to load and unload passengers on the streets instead of inside the terminals, then reasonable specifications for the size of terminals could be instituted, with permits to operate the same denied those which are unable to meet the specifications. In the subject ordinances, however, the scope of the proscription against the maintenance of terminals is so broad that even entities which might be able to provide facilities better than the franchised terminal are barred from operating at all. The Court is not unaware of the resolutions of various barangays in Lucena City supporting the establishment of a common terminal, and similar expressions of support from the private sector, copies of which were submitted to this Court by Lucena Grand Central Terminal, Inc. The weight of popular opinion, however, must be balanced with that of an individuals rights. Torts and Damages Element Quasi Delicts David Taylor was a 15 year old boy who spent time as a cabin boy at sea; he was also able to learn some principles of mechanical engineering and mechanical drawing from his dads office (his dad was a mechanical engineer); he was also employed as a mechanical draftsman earning P2.50 a day all said, Taylor was mature well beyond his age. One day in 1905, he and another boy entered into the premises of Manila Electric power plant where they found 20-30 blasting caps which they took home. In an effort to explode the said caps, Taylor experimented until he succeeded in opening the caps and then he lighted it using a match which resulted to the explosion of the caps causing severe injuries to his companion and to Taylor losing one eye. Taylor sued Manila Electric alleging that because the company left the caps exposed to children, they are liable for damages due to the companys negligence. ISSUE: Whether or not Manila Electric is liable for damages. HELD: No. The SC reiterated the elements of quasi delict as follows: (1) Damages to the plaintiff. (2) Negligence by act or omission of which defendant personally, or some person for whose acts it must respond, was guilty.

Section 3(a) of R.A. No. 6395, as amended, and the implementing rule of R.A. No. 8974 indeed state that only 10% of the market value of the property is due to the owner of the property subject to an easement of right-of-way, said rule is not binding on the Court. Well-settled is the rule that the determination of just compensation in eminent domain cases is a judicial function. Categories: Constitutional Law 1, G.R. No. 160725 LUCENA GRAND CENTRAL TERMINAL, INC. v. JAC LINER, INC. 452 SCRA 174 (2005), EN BANC (Carpio Morales, J.) The true role of Constitutional law is to effect an equilibrium between authority and liberty so that rights are exercised within the framework of the law and the laws are enacted with due deference to rights. FACTS: Two ordinances were enacted by the Sangguniang Panlungsod of Lucena with the objective of alleviating the traffic congestion said to have been caused by the existence of various bus and jeepney terminals within the city. City Ordinance 1631 grants franchise to the Lucena Grand Central Terminal, Inc. to construct, finance, establish, operate and maintain common bus- jeepney terminal facility in the City of Lucena. City Ordinance 1778, on the other hand, strips out all the temporary terminals in the City of Lucena the right to operate which as a result favors only the Lucena Grand Central Terminal, Inc. The Regional Trial Court of Lucena declared City Ordinance 1631 as a valid excercise of police power while declaring City Ordinance 1778 as null and void for being invalid. Petitioner Lucena Grand Central Terminal, Inc. filed its Motion for Reconsideration which was denied. Lucena then elevated it via petition for review under Rule 45 before the Court. The Court referred the petition to the Court of Appeals (CA) with which it has concurrent jurisdiction. The CA dismissed the petition and affirmed the challenged orders of the trial court. Its motion for reconsideration having been denied by the CA, Lucena now comes to the Court via petition for review to assail the Decision and Resolution of the CA. ISSUE: Whether or not the means employed by the Lucena Sannguniang Panlungsod to attain its professed objective were reasonably necessary and not duly oppressive upon individuals HELD: With the aim of localizing the source of traffic congestion in the city to a single location, the subject ordinances prohibit the operation of all bus and jeepney terminals within Lucena, including those already existing, and allow the operation of only one common terminal located outside the city proper, the franchise for which was granted to Lucena. The common carriers plying routes to

(3) The connection of cause and effect between the negligence and the damage. In the case at bar, it is true that Manila Electric has been negligent in disposing off the caps which they used for the power plant, and that said caps caused damages to Taylor. However, the causal connection between the companys negligence and the injuries sustained by Taylor is absent. It is in fact the direct acts of Taylor which led to the explosion of the caps as he even, in various experiments and in multiple attempts, tried to explode the caps. It is from said acts that led to the explosion and hence the injuries. Taylor at the time of the accident was well-grown youth of 15, more mature both mentally and physically than the average boy of his age; he had been to sea as a cabin boy; was able to earn P2.50 a day as a mechanical draftsman thirty days after the injury was incurred; and the record discloses throughout that he was exceptionally well qualified to take care. The evidence of record leaves no room for doubt that he well knew the explosive character of the cap with which he was amusing himself. The series of experiments made by him in his attempt to produce an explosion admit of no other explanation. His attempt to discharge the cap by the use of electricity, followed by his efforts to explode it with a stone or a hammer, and the final success of his endeavors brought about by the applications of a match to the contents of the cap, show clearly that he knew what he was about. Nor can there be any reasonable doubt that he had reason to anticipate that the explosion might be dangerous. The just thing is that a man should suffer the damage which comes to him through his own fault, and that he cannot demand reparation therefor from another. GESTOPA VS. CA FACTS- ACCEPTANCE IN DONATION Acceptance is a mark that the donation is inter vivos. Donations mortis causa, being in the form of a will, are not required to be accepted by the donee during the donors lifetime. FACTS: Spouses Danlag own six parcels of land. To four parcels of land, they executed a donation mortis causa in favor of respondent Mercedes Danlag-Pilapil, reserving donor's rights to amend, cancel, or revoke the donation and to sell or encumber such properties. Years later, they executed another donation, this time inter vivos, to six parcels of land in favor of respondents, reserving their rights to the fruits of the land during their lifetime and for prohibiting the donee to sell or dispose the properties donated. Subsequently, the spouses sold 2 parcels to herein petitioners, spouses Gestopa, and eventually revoking the donation. Respondent filed a petition to quiet title, stating that she had already become the owner of the parcels of land. Trial Court ruled in favor of petitioners, but CA reversed. ISSUE: Whether the (second) donation was inter vivos or mortis causa RULING: It was donation inter vivos. The spouses were aware of the difference between the two donations, and that they needed to execute another deed of donation inter vivos, since it has a different application to a donation mortis causa. Also, the court stated four reasons to the matter: (1) that the spouses donated the parcels of land out of love and affection, a clear indication of a donation inter vivos; (2) the reservation of a lifetime usufruct; (3) reservation of sufficient properties for maintenance that shows the intention to part with their six lot; and (4) respondent's acceptance, contained in the deed of donation. Once a

deed of donation has been accepted, it cannot be revoked, except for officiousness or ingratitude, which the spouses failed to invoke. REPUBLIC OF THE PHILIPPINES V. LEON SILIM AND ILDEFONSA MANGUBAT (2001) Silim and Mangubat donated a parcel of land in favor of the Bureau of Public Schools, Municipality of Malangas, Zamboanga Del Sur, on the condition that said land shall be used exclusively and forever for educational purposes. However, as the said parcel of land was too small for the construction of the planned Bagong Lipunan School Building, the Province of Zamboanga, through its division superintendent, executed a deed of exchange in favor of a new and suitable location. Consequently, whatever was built on the donated lot was dismantled and transferred to the new fitting location. The issue arose when the donators found out that Vice-Mayor Wilfredo Palma took the land for himself and constructed his house thereon. Silim and Mangubat sought to have the donation declared null and void on the ground that the condition of the donation was violated. ISSUE: Whether or not the donation was valid in view of the fact that the school, which it was conditioned on, was never built thereon. RULING: YES. The Court upheld the validity of the donation. Firstly, it ruled that there was a valid acceptance in accordance with Arts. 745 and 749 of the Civil Code: it was made personally or through an agent, and it was made in a public document. Anent the second contention concerning the non-fulfillment of the consideration, the Court ruled that contrary to Silim and Mangubats argument, the parcel of land was used precisely for school purposes. The exchange of the donated lot for a bigger lot was done in pursuance with the condition that they (Silim and Mangubat) imposed. Remember: the lot was exchanged with a bigger lot to give way for the construction of Bagong Lipunan Elementary School and for no other reason. The exclusivity of the purpose was not altered or affected. In fact, the exchange of the lot for a much bigger one was in furtherance and enhancement of the purpose of the donation. The acquisition of the bigger lot paved the way for the release of funds for the construction of Bagong Lipunan school building which could not be accommodated by the limited area of the donated lot. BONIFACIA MATEO vs. GERVACIO LAGUA October 30, 1969 FACTS: Sometime in 1917, the parents of Alejandro Lagua donated two lots to him in consideration of his marriage to petitioner Bonifacia Mateo. The marriage was celebrated on May 15, 1917 and thereafter the couple took possession of the lots, but the certificates of title remained in the donors name. In 1923, Alejandro died, leaving behind his widow Bonifacia with their infant daughter, who lived with the father-in-law Cipriano Lagua who in turn undertook to farm on the donated lots. At first, Cipriano gave to Bonifacia the share from the lots harvests, but in 1926 he refused to deliver to petitioner the said share, which reason prompted her to initiate an action and won for her possession of the lots plus damages. On July 31, 1941, Cipriano executed a deed of sale of the said lots in favor of his younger son, herein respondent Gervacio. Petitioner learned of this only in 1956 when Cipriano stopped giving to petitioner her share to the harvest.

A Transfer Certificate of Title (TCT) was issued under respondents name by the Registry of Deeds (ROD) of Pangasinan. The CFI of Pangasinan declared the TCT issued to respondent null and void and ordered cancelled by the ROD, and for respondent to vacate and deliver the lots to petitioner. In 1957, Gervacio and Cipriano filed with the CFI for the annulment of the donation of the two lots. While the case was pending, Cipriano died in 1958. It was dismissed for prescription, having been filed after the lapse of 41 years. When appealed, the CA in 1966 held that the donation to Alejandro of the two lots with the combined area of 11,888 sq. m. exceeded by 494.75 sq. m. his legitime and the disposable portion that Cipriano could have freely given by will, and to the same extent prejudiced the legitime of Ciprianos other heir, Gervacio. The donation was thus declared inofficious and herein petitioners were ordered to reconvey to Gervacio a portion of 494.75 sq. m. from any convenient part of the lots. ISSUE: Whether or not the Court of Appeals correctly reduced the donation propter nuptias for being inofficious. HELD: Decision of CA based on unsupported assumptions set aside; trial courts order of dismissal sustained. Before the legal share due to a compulsory heir may be reached, the net estate of the decedent must be ascertained, by deducting all payable obligations and charges from the value of the property owned by the deceased at the time of his death; then, all donations subject to collation would be added to it. With the partible estate thus determined, the legitimes of the compulsory heirs can be established, and only thereafter can it be ascertained whether or not a donation had prejudiced the legitimes. Certainly, in order that a donation may be reduced for being inofficious, there must be proof that the value of the donated property exceeds that of the disposable free portion plus the donees share as legitime in the properties of the donor. In the present case, it can hardly be seen that, with the evidence then before the court, it was in any position to rule on the inofficiousness of the donation involved here, and to order its reduction and reconveyance of the deducted portion to the respondents. Article 908. To determine the legitime, the value of the property left at the death of the testator shall be considered, deducting all debts and charges, which shall not include those imposed in the will. To the value of the hereditary estate, shall be added the value of all donations by the testator that are subject to collation, at the time he made them. CASE: QUILALA vs. ALCANTARA TOPIC: MAKING ANG ACCEPTANCE OF DONATIONG.R No.: 132681 December 3, 2001FACTS:On February 20, 1981, Catalina Quilala (donor) executed a"Donation of Real Property Inter Vivos" in favor of VioletaQuilala (donee) over a parcel of land located in Sta. Cruz,Manila and registered in her name.The "Donation of Real Property Inter Vivos" consists of twopages. The first page contains the deed of donation itself, andis signed on the bottom portion by

Catalina Quilala andVioleta Quilala, and two instrumental witnesses. The secondpage contains the Acknowledgment, which states merely thatCatalina Quilala personally appeared before the notary publicand acknowledged that the donation was her free andvoluntary act and deed. There appear on the left-hand marginof the second page the signatures of Catalina Quilala and oneof the witnesses, and on the right-hand margin the signaturesof Violeta Quilala and the other witnessOn November 7, 1983, Catalina Quilala died. Violeta Quilalalikewise died on May 22, 1984. Petitioner Ricky Quilalaalleges that he is the surviving son of Violeta Quilala.Meanwhile, respondents Gliceria Alcantara, LeonoraAlcantara, Ines Reyes and Juan Reyes, claiming to beCatalina's only surviving relatives within the fourth civildegree of consanguinity instituted an action for thedeclaration of nullity of the donation inter vivos , and for thecancellation of the TCT in the name of Violeta Quilala.The trial court rendered a decision declaring null and void thedeed of donation of real property inter vivos executed byCatalina Quilala in favor of Violeta Quilala. The trial courtfound that since it was acknowledged before a notary publiconly by the donor, Catalina, there was no acceptance byVioleta of the donation in a public instrument. The decisionwas affirmed by the CA.ISSUE: Whether or not the donation executed by Catalina infavor of Violeta is validHELD: valid even if the acknowledgment was only signed by the donor Below the terms and stipulations of the donation, the donor,donee and their witnesses affixed their signature. However,the Acknowledgment appearing on the second pagementioned only the donor, Catalina Quilala. Thus, the trialcourt ruled that for Violeta's failure to acknowledge heracceptance before the notary public, the same was set forthmerely on a private instrument, i.e., the first page of theinstrument.We disagree.As provided for in Section 112, paragraph 2 of PD No. 1529,the second page of the deed of donation, on which theAcknowledgment appears, was signed by the donor and onewitness on the left-hand margin. The donee and the otherwitness signed on the right hand margin. Surely, therequirement that the contracting parties and their witnessesshould sign on the left-hand margin of the instrument is notabsolute. The intendment of the law merely is to ensure thateach and every page of the instrument is authenticated bythe parties. The requirement is designed to avoid thefalsification of the contract after the same has already beenduly executed by the parties. Hence, a contracting partyaffixes his signature on each page of the instrument to certifythat he is agreeing to everything that is written thereon atthe time of signing.Simply put, the specification of the location of the signatureis merely directory . The fact that one of the parties signs onthe wrong side of the page does not invalidate the document.In the same vein, the lack of an acknowledgment by thedonee before the notary public does not also render thedonation null and void. The instrument should be treated inits entirety. It cannot be considered a private document inpart and a public document in another part . The fact that itwas acknowledged before a notary public converts the deedof donation in its entirety a public instrument. The fact thatthe donee was not mentioned by the notary public in theacknowledgment is of no moment. To be sure, it is theconveyance that should be acknowledged as a free andvoluntary act.

También podría gustarte